Location via proxy:   [ UP ]  
[Report a bug]   [Manage cookies]                

Polity Module 1 - Part 2

Download as pdf or txt
Download as pdf or txt
You are on page 1of 81

YWM5YTIyN2MzNDVk

ILP-2022
(Integrated Learning Program)

Value Add Notes (VAN)

POLITY
YWM5YTIyN2MzNDVk

IASBABA’S ILP 2022 – POLITY MODULE 1

FUNDAMENTAL RIGHTS

Rights are legal, social, or ethical principles of freedom or entitlement; that is, rights are the
fundamental normative rules about what is allowed of people or owed to people, according
to some legal system, social convention, or ethical theory. Rights are of essential importance
in such disciplines as law and ethics, especially theories of justice and deontology.

Natural rights are rights which are "natural" in the sense of "not artificial, not man-made", as
in rights deriving from human nature or from the edicts of a god. They are universal; that is,
they apply to all people, and do not derive from the laws of any specific society. They exist
necessarily, inhere in every individual, and can't be taken away.

For example, it has been argued that humans have a natural right to life. These are sometimes
called moral rights or inalienable rights.

Legal rights, in contrast, are based on a society's customs, laws, statutes or actions by
legislatures. An example of a legal right is the right to vote of citizens. Citizenship, itself, is
often considered as the basis for having legal rights and has been defined as the "right to have
rights". Legal rights are sometimes called civil rights or statutory rights and are culturally and
politically relative since they depend on a specific societal context to have meaning.

Q. Right to vote and to be elected in India is a (2017)

a) Fundamental Right
b) Natural Right
c) Constitutional Right
d) Legal Right

Fundamental Rights are protected and Guaranteed by the Constitution and they cannot be
taken away by an ordinary law enacted by the legislature. If a legal right of a person is violated,
he can move to an ordinary court, but if a fundamental right is violated the Constitution
provides that the affected person may move to High court or Supreme Court.

UNIVERSAL DECLARATION OF HUMAN RIGHTS


The General Assembly, Proclaims the Universal Declaration of Human Rights as a common
standard of achievement for all peoples and all nations, to the end that every individual and
every organ of society, keeping this Declaration constantly in mind, shall strive by teaching
and education to promote respect for these rights and freedoms and by progressive
measures, national and international, to secure their universal and effective recognition and
observance, both among the peoples of Member States themselves and among the peoples
of territories under their jurisdiction.

WWW.IASBABA.COM 1
YWM5YTIyN2MzNDVk

IASBABA’S ILP 2022 – POLITY MODULE 1

Q. Consider the following:

1. Right to education
2. Right to equal access to public service
3. Right to food
Which of the above is/are Human Right/Human Rights under “Universal Declaration of
Human Rights”?

a) 1 only
b) 1 and 2 only
c) 3 only
d) 1, 2 and 3

In 1925 the Indian National Congress finalized the draft of Common Wealth of India Bill
adopting a 'Declaration of Rights.' The Madras Session of the Congress held in the year 1927
– demanded incorporation of a 'Declaration of Fundamental Rights' in any future
constitutional framework.

The Karachi Session of the Congress in 1931 adopted a detailed programme of fundamental
rights. The Government of India Act, 1935 was passed without any bill of rights much to the
disappointment of the Indian leaders.

It was the 'Sapru Committee' of 1945 that subsequently stressed the need for a written code
of fundamental rights and the Constituent Assembly raised a forceful demand for the
inclusion of human rights in the Constitution.

The framers of the Constitution derived inspiration from the Constitution of USA (i.e., Bill of
Rights) and adopted Fundamental Rights in Part III of the Constitution from Articles 12 to 35.

Part III of the Constitution (which contains FRs) is rightly described as the ‘Magna Carta of
India’.

Fundamental Rights are justiciable - allows persons to move the courts for their
enforcement, if and when they are violated. The Fundamental Rights are named so because
they are guaranteed and protected by the Constitution, which is the fundamental law of the
land.

Fundamental Rights are guaranteed by the Constitution to all persons and legal entities
without any discrimination. They uphold the equality of all individuals, the dignity of the
individual, the larger public interest and unity of the nation.

WWW.IASBABA.COM 2
YWM5YTIyN2MzNDVk

IASBABA’S ILP 2022 – POLITY MODULE 1

ESTABLISHING ‘A GOVERNMENT OF LAWS AND NOT OF MEN’


The Constitution listed the rights that would be specially protected and called them
‘fundamental rights’.

These rights are so important that the Constitution has separately listed them and made
special provisions for their protection. The Constitution itself ensures that they are not
violated by the government.

They promote the ideal of political democracy. They prevent the establishment of an
authoritarian and despotic rule in the country and protect the liberties and freedoms of the
people against the invasion by the State.

They check and balance the tyrannical power of executive and arbitrary law of legislature. In
short, they aim at establishing ‘a government of laws and not of men.

ORDINARY RIGHTS AND FUNDAMENTAL RIGHTS:


Rights may be either statutory or constitutional. When a right is enacted in an ordinary law of
the land it is a statutory right. And when the Constitution guarantees any right it assumes the
status of constitutional right.

Ordinary legal rights are protected and enforced by ordinary law, Fundamental Rights are
protected and guaranteed by the Constitution of the country.

Ordinary rights may be changed by the legislature by ordinary process of law making, but a
fundamental right may only be changed by amending the Constitution itself.

Judiciary has the powers and responsibility to protect the fundamental rights from violations
by actions of the government. Executive as well as legislative actions can be declared illegal
by the judiciary if these violate the Fundamental rights or restrict them in an unreasonable
manner.

Originally, the Constitution provided for seven Fundamental Rights viz,

• Right to equality (Articles 14–18)


• Right to freedom (Articles 19–22)
• Right against exploitation (Articles 23–24)
• Right to freedom of religion (Articles 25–28)
• Cultural and educational rights (Articles 29–30)
• Right to property (Article 31)
• Right to constitutional remedies (Article 32)
However, the right to property was deleted from the list of Fundamental Rights by the 44th
Amendment Act, 1978.

It is made a legal right under Article 300-A in Part XII of the Constitution. So, at present, there
are only six Fundamental Rights.

WWW.IASBABA.COM 3
YWM5YTIyN2MzNDVk

IASBABA’S ILP 2022 – POLITY MODULE 1

FUNDAMENTAL RIGHTS AT A GLANCE

Note: Try to remember according to these divisions. Questions have been asked from here.

Make some story/trick to remember. You can also share a good trick/story to remember this
with your friends here.

WWW.IASBABA.COM 4
YWM5YTIyN2MzNDVk

IASBABA’S ILP 2022 – POLITY MODULE 1

FEATURES
1. Some of them are available only to the citizens while others are available to all persons
whether citizens, foreigners or legal persons like corporations or companies.
2. The state can impose restrictions on Fundamental rights. (They are not absolute but
qualified).
3. Except Fundamental rights guaranteed under Articles 20 and 21 remaining
Fundamental rights can be suspended during operation of National Emergency.
4. Article 19 can be suspended only when emergency is declared on the grounds of war
or external aggression and not on the grounds of armed rebellion.
5. Most of the FRs are available against the arbitrary action of the State.
6. Some of FRs are negative in character, they place limitations on the authority of the
State
7. Some other FRs are positive in nature, as they confer certain privileges on the persons.
8. FRs are defended and guaranteed by the Supreme Court. Hence, the aggrieved
person can directly go to the Supreme Court, not necessarily by way of appeal against
the judgment of the high courts.
9. FRs are not sacrosanct or permanent --> Meaning, the Parliament can curtail or repeal
them. But only by a constitutional amendment act and not by an ordinary act.
Moreover, this can be done without affecting the ‘basic structure’ of the Constitution.
10. Their scope of operation is limited by Article 31A (saving of laws providing for
acquisition of estates, etc.), Article 31B (validation of certain acts and regulations
included in the 9th Schedule) and Article 31C (saving of laws giving effect to certain
directive principles).
11. Article 33 enables the parliament to restrict the application of FR to the members of
armed forces, para-military forces, police forces, intelligence agencies and analogous
services.
12. Most of them are directly enforceable (self-executory) while a few of them can be
enforced on the basis of a law made for giving effect to them. Such a law can be made
only by the Parliament and not by state legislatures so that uniformity throughout the
country is maintained (Article 35).

ARTICLE 12 PROVIDES THE ‘DEFINITION OF THE STATE’.

The “State” includes –

• the Government and Parliament of India


• the government and the state legislature
• all local authorities (municipalities, Panchayati Raj, District boards. Etc)
• Other statutory and non-statutory authorities (LIC, ONGC etc.).

WWW.IASBABA.COM 5
YWM5YTIyN2MzNDVk

IASBABA’S ILP 2022 – POLITY MODULE 1

The actions of the state (all the above said) can be challenged in the courts as the violation of
Fundamental Rights.

According to the Supreme Court, even a private body or an agency working as an instrument
of the State falls within the meaning of the ‘State’ under Article 12.

Think

• Article 12 does not explicitly mention judiciary. However, as judiciary is the guardian
of fundamental rights, an obvious question that can arise is ‘Can Courts be defined as
‘State’ under Article 12 of the Indian Constitution?’
The High Court of Bombay recently answered this question in the case of The National
Federation of the Blind, Maharashtra & Anr v. The High Court of Judicature of Bombay,
wherein it held that ‘Courts are included within the definition of “State” only on the
administrative side while dealing with employees or while taking decisions in
administrative capacity and not on the judicial side’.

ARTICLE 13 – LAWS INCONSISTENT WITH OR IN DEROGATION OF THE FUNDAMENTAL


RIGHTS.

Article 13 declares that all laws that are inconsistent with or in derogation of any of the
fundamental rights shall be void.

It provides for the “doctrine of judicial review”.

Supreme Court (according to Article 32) and the high courts (according to Article 226) can
declare a law unconstitutional and invalid on the ground of contravention of any of the
Fundamental Rights.

The term ‘law’ in Article 13 has been given a wide connotation so as to include the following:

a) Permanent laws enacted by the Parliament or the state legislatures;


b) Temporary laws like ordinances issued by the president or the state governors;
c) Statutory instruments in the nature of delegated legislation (executive legislation) like
order, bye-law, rule, regulation or notification; and
d) Non-legislative sources of law, that is, custom or usage having the force of law

Article 13 declares that a constitutional amendment is not a law and hence cannot be
challenged (24th amendment act).

However, the Supreme Court held in the Kesavananda Bharati case (1973) that a
Constitutional amendment can be challenged on the ground that it violates a fundamental
right that forms a part of the ‘basic structure’ of the Constitution and hence, can be declared
as void.

Please note: The word “Judicial Review” is nowhere mentioned in the Constitution.

WWW.IASBABA.COM 6
YWM5YTIyN2MzNDVk

IASBABA’S ILP 2022 – POLITY MODULE 1

ARTICLE 14-18 RIGHT TO EQUALITY (14-18)

ARTICLE 14: EQUALITY BEFORE LAW AND EQUAL PROTECTION OF LAWS

Article 14 says that State shall not deny to any person equality before the law or the equal
protection of the laws within the territory of India.

The article 14 aims to establish the "Equality of Status and Opportunity" as embodied in the
Preamble of the Constitution.

Equality before law no one is above the law of the land

Equality before law: The absence of any special privileges in favor of any person. This concept
ensures the following,

a) The absence of any special privileges in favor of any person,


b) The equal subjection of all persons to the ordinary law of the land administered by
ordinary law courts, and
c) No person (whether rich or poor, high or low, official or non-official) is above the law.

Note: Equality before law is taken from the British Constitution.

Equal Protection of Laws: The equality of treatment under equal circumstances.

a) The equality of treatment under equal circumstances, both in the privileges conferred
and liabilities imposed by the laws,
b) The similar application of the same laws to all persons who are similarly situated, and
c) The like should be treated alike without any discrimination.

Note: Equal Protection of Laws is taken from the American Constitution.

The concept of ‘equal protection of the laws’ requires the State to give special treatment to
persons in different situations in order to establish equality amongst all. It is positive in
character.

Therefore, the necessary corollary to this would be that equals would be treated equally,
whilst un-equals would have to be treated unequally.

These provisions confer rights on all persons whether citizens or foreigners.

The word ‘person’ includes legal persons, viz, statutory corporations, companies, registered
societies or any other type of legal person.

The Supreme Court held that the ‘Rule of Law’ as embodied in Article 14 is a ‘basic feature’ of
the constitution. Hence, it cannot be destroyed even by an amendment.

WWW.IASBABA.COM 7
YWM5YTIyN2MzNDVk

IASBABA’S ILP 2022 – POLITY MODULE 1

RULE OF LAW

The guarantee of equality before the law is an aspect of what Dicey calls the Rule of Law in
England. It means that no man is above the law and that every person whatever be his rank
or condition is subject to the jurisdiction of ordinary courts.

Rule of law require that no person shall be subjected to harsh, uncivilized or discriminatory
treatment even when the object is the securing of the paramount exigencies of law and order.

Professor Dicey gave three meanings of the Rule of Law

1. Absence of arbitrary power or supremacy of the law

It means the absolute supremacy of law as opposed to the arbitrary power of the
Government. In other words-a man may be punished for a breach of law, but he can’t be
punished for anything else.

2. Equality before law

It means subjection of all classes to the ordinary law of land administrated by ordinary law
courts. This means that no one is above law all are equal in eyes of law

3. Absence of individual liberty

There are various constitutions that provide individual liberty but not provide method It
means that the source of the right of individuals is not the written constitution. U.K. doesn’t
have provision for individual liberty.

RULE OF LAW IN INDIA

1. Supremacy of Law:

The First meaning of the Rule of Law is that 'no man is punishable or can lawfully be made to
suffer in body or goods except for a distinct breach of law established in the ordinary legal
manner before the ordinary courts of the land. It implies that a man may be punished for a
breach of law but cannot be punished for anything else. No man can be punished except for
a breach of law. An alleged offence is required to be proved before the ordinary courts in
accordance with the ordinary procedure.

2. Equality before Law:

The Second meaning of the Rule of Law is that no man is above law. Every man whatever be
his rank or condition is subject to the ordinary law of the realm and amenable to the
jurisdiction of the ordinary tribunals. Everybody under Article 14 is equal before law and has
equal protection.

WWW.IASBABA.COM 8
YWM5YTIyN2MzNDVk

IASBABA’S ILP 2022 – POLITY MODULE 1

3. Individual Liberty

Lot of individual liberty is mention like fundamental right in Article 21- protection of life and
personal liberty, article 19- Right to freedom etc. and courts are there to protect individual
liberty.

The first and second aspect applies to Indian system but the third aspect of the Dicey’s rule
of law does not apply to Indian system as the source of right of individuals is the constitution
of India.

The constitution is the supreme law of the land and all laws passed by the legislature must be
consistent with provisions of the constitution

The rule of law imposes a duty upon state to take special measure to prevent and punish
brutality by police methodology. The rule of law embodied in article 14 is the basic feature of
the Indian constitution and hence it can’t be destroyed even by an amendment of the
constitution under article 368 of the constitution.

EXCEPTIONS TO EQUALITY

The rule of equality before law is not absolute and there are constitutional and other
exceptions to it. These are mentioned below:

• President or Governor of state is not answerable to court of law for exercising their
executive powers.
• No criminal proceeding against President or Governor of state can be instituted or
continued during their tenure in office.
• No civil proceeding in which there is a claim of compensation can be instituted against
President or Governor of state except after the expiry of 2-month notice issued against
them.
• Under international law, foreign diplomats who are on a visit to India or posted here,
and leaders or heads of state on their official visit are not answerable in the local
courts.

ARTICLE 15: PROHIBITION OF DISCRIMINATION ON GROUNDS OF RELIGION, RACE, CASTE,


SEX OR PLACE OF BIRTH

Article 15 prohibits the state from discriminating any citizen on ground of any religion, race,
caste, sex, and place of birth or any of them. It provides that there shall be no restriction on
any person on any of the above bases to access and use the public places.

WWW.IASBABA.COM 9
YWM5YTIyN2MzNDVk

IASBABA’S ILP 2022 – POLITY MODULE 1

EXCEPTIONS

1. However, this Article does not prevent the State from making any special provisions
for women or children.

For example, reservation of seats for women in local bodies or provision of free education for
children.

2. It also allows the State to extend special provisions for socially and educationally
backward classes for their advancement. It applies to the Scheduled Castes (SC) and
Scheduled Tribes (ST) as well.

For example, reservation of seats or fee concessions in public educational institutions.

3. The state is empowered to make any special provision for the advancement of any
socially and educationally backward classes of citizens or for the scheduled castes or
the scheduled tribes regarding their admission to educational institutions including
private educational institutions, whether aided or unaided by the state, except the
minority educational institutions (93rd Amendment Act of 2005).

In order to give effect to this provision, the Centre enacted the Central Educational
Institutions (Reservation in Admission) Act, 2006, providing a quota of 27% for candidates
belonging to the Other Backward Classes (OBCs) in all central higher educational institutions
including the Indian Institutes of Technology (IITs) and the Indian Institutes of Management
(IIMs).

Think!

• Creamy layer

In order to bring about the real equality, preference given to the socially and educationally a
disadvantaged group is justified.

EQUALITY OF OPPORTUNITY IN MATTER OF PUBLIC EMPLOYMENT

ARTICLE 16 PROVIDES THE GUARANTEE OF EQUALITY OF OPPORTUNITY IN MATTERS OF


PUBLIC EMPLOYMENT.

Article 16(1) and 16(2) have laid down a general rule that there shall be equal opportunity for
all citizens and thus emphasizes on universality of Indian Citizenship.

No citizen can be discriminated against or be ineligible for any employment or office under
the State on grounds of only religion, race, caste, sex, descent, place of birth or residence.

WWW.IASBABA.COM 10
YWM5YTIyN2MzNDVk

IASBABA’S ILP 2022 – POLITY MODULE 1

EXCEPTIONS
As per Article 16(3) residence qualifications may be made necessary in the case of
appointments under the state for particular positions, thus making the domicile provisions
stronger, however, the power is not vested in the states but in Parliament to prescribe the
requirement as to residence in the state.

Article 16(4) empowers the state to make special provisions for the reservation of
appointments or posts in favour of any “backward class of citizens” which in the opinion of
state are not adequately represented in the services of the state.

Equality, as guaranteed in our Constitution, not only conceives of providing formal equality
but also to provide for real and absolute equality. Articles 14 and 15(1) enable and
contemplate classification to achieve the Constitutional Objective of real equality. Articles
15(4) and 16(4) flows out of Articles 15(1) and 16(1) respectively and can never be considered
as exceptions to Article 15(1) and Article 16(1)

Equal pay for equal work:

The principle of equal pay for equal work has not been specifically declared to be a
Fundamental Right under the Indian Constitution. But it certainly is a constitutional goal. The
principle of equal pay for equal work would be an abstract doctrine not attracting Article 1 4
if sought to be applied to them.

But equality clause will have some substance if equal work means equal pay and such right is
deducible from Article 14 and 16 in the light of Preamble to the Constitution and Article
39(d) of the Directive Principles of the Constitution.

There cannot be any unequal scale of pay on the basis of no classification or irrational
classification when they do identical work under the same employer. Thus, the court makes
a liberal use of the equality clause to make the directive of equal pay for equal work more
authentically constitutional than what it is.

Constitution (One Hundred and Third Amendment) Act, 2019

• It introduces 10% reservation for Economically Weaker Sections (EWS) of society for
admission to Central Government-run educational institutions and private
educational institutions (except for minority educational institutions), and for
employment in Central Government jobs.
• The economic reservation was introduced in the Constitution by amending Articles 15
and 16 and adding clauses empowering the State governments to provide reservation
on the basis of economic backwardness.
• The act inserted Articles 15(6) and 16(6) of the Constitution that provided for these
reservations.
• Article 46 of the Constitution (DPSP) enjoins that the State shall promote with special
care the educational and economic interests of the weaker sections of the people.

WWW.IASBABA.COM 11
YWM5YTIyN2MzNDVk

IASBABA’S ILP 2022 – POLITY MODULE 1

• Supreme Court referred to a five-judge Bench the “substantial question of law”


whether grant of 10% reservation to economically weaker sections of the society is
unconstitutional and violates the 50% ceiling cap on quota declared by the court itself.
• SC will decide whether “economic backwardness” can be the sole criterion for granting
quota in government jobs and educational institutions for those who would otherwise
have to compete in the general category.
• The petitioners challenged the validity of the Constitutional Amendment, saying the
50% quota limit was part of the Basic Structure of the Constitution.
Reservation for Disabled
Supreme Court confirmed that persons suffering from disabilities are also socially backward
and entitled to the same benefits of relaxation as Scheduled Caste/Scheduled Tribe
candidates in public employment and education.
The Bench led by Justice Rohinton F. Nariman confirmed that 3% reservation should be given
to disabled persons both in direct recruitment and in promotions.
A perusal of Indra Sawhney would reveal that the ceiling of 50% reservation applies only to
reservation in favour of Other Backward Classes under Article 16(4) of the Constitution of
India whereas the reservation in favour of persons with disabilities is horizontal, which is
under Article 16(1) of the Constitution.
Right to reservation is not a Fundamental Right

• All political parties from Tamil Nadu had filed a writ petition in the Supreme Court
seeking direction to the Centre to implement 50% OBC reservation in the all-India
NEET seats surrendered by the state.
• They have accused the Centre of “violating the right of the people to have a fair
education” by neither implementing the 50% quota for OBC in Tamil Nadu nor
providing 27% reservation for OBC candidates in other states for the All India Quota
seats.
• The Court observed that the Right to Reservation is not a fundamental right.
• In February 2020, the Supreme Court ruled that there is no fundamental right to claim
reservation in public jobs and no court can order a state government to provide for
reservation to SC/STs.
• Article 32 is available only for violation of fundamental rights, but right to reservation
is not a fundamental right.
Overseas Citizen of India (OCI)

• Union government has told the Delhi High Court that OCI cardholders do not enjoy
fundamental rights guaranteed by the Constitution, including the right to freedom of
speech and expression.

WWW.IASBABA.COM 12
YWM5YTIyN2MzNDVk

IASBABA’S ILP 2022 – POLITY MODULE 1

ABOLITION OF UNTOUCHABILITY

ARTICLE 17 ABOLISHES THE UNTOUCHABILITY AND ITS PRACTICE IN ANY FORM IS MADE
PUNISHABLE UNDER THE LAW.

This was the article which was adopted with the cries of "Mahatma Gandhi ki Jai".

Note: The term ‘untouchability’ has not been defined either in the Constitution or in the Act.

Accordingly, the Parliament passed Untouchability (Offences) Act, 1955.

In the year 1976, this act is renamed as Civil Rights Act, 1955.

Under the Protection of Civil Rights Act (1955), the offences committed on the ground of
untouchability are punishable either by imprisonment up to six months or by fine up to Rs500
or both.

The act defines civil right as any right accruing to a person by reason of the abolition of
untouchability by Article 17 of the Constitution.

A person convicted of the offence of ‘untouchability’ is disqualified for election to the


Parliament or state legislature.

The Supreme Court held that the right under Article 17 is available against private individuals
and it is the constitutional obligation of the State to take necessary action to ensure that this
right is not violated

Think!

• How Article 17 is related and debated in the recent case of Sabarimala Temple entry
w.r.t Women?

ARTICLE 18 -- ABOLITION OF TITLES EXCEPT MILITARY AND ACADEMIC

Article 18 prevents the state from confirming any title except military and academic
distinction.

Article 18 prohibits the Indian citizens from receiving titles from any foreign state.

Article 18 abolishes titles and makes four provisions in that regard:

a) It prohibits the state from conferring any title (except a military or academic
distinction) on anybody, whether a citizen or a foreigner.
b) It prohibits a citizen of India from accepting any title from any foreign state.
c) A foreigner holding any office of profit or trust under the state cannot accept any title
from any foreign state without the consent of the president.

WWW.IASBABA.COM 13
YWM5YTIyN2MzNDVk

IASBABA’S ILP 2022 – POLITY MODULE 1

d) No citizen or foreigner holding any office of profit or trust under the State is to accept
any present, emolument or office from or under any foreign State without the consent
of the president.

Note the Supreme Court upheld the constitutional validity of the National Awards—Bharat
Ratna, Padma Vibhushan, Padma Bhushan and Padma Sri. It ruled that these awards do not
amount to ‘titles’.

ARTICLE 19-22: RIGHT TO FREEDOM

ARTICLE 19 -- PROTECTION OF CERTAIN RIGHTS REGARDING FREEDOM OF SPEECH, ETC.


Article 19 guarantees to all citizens the six rights. These are:

1. Right to freedom of speech and expression.

Why to protect freedom of speech?

Freedom of speech offers human being to express his feelings to other, but this is not the only
reason; purpose to protect the freedom of speech. There could be more reasons to protect
these essential liberties. There are four important justifications for freedom of speech –

a. For the discovery of truth by open discussion - According to it, if restrictions on


speech are tolerated, society prevents the ascertainment and publication of accurate
facts and valuable opinion. That is to say, it assists in the discovery of truth.
b. Free speech as an aspect of self- fulfillment and development – freedom of speech is
an integral aspect of each individual’s right to self-development and self-fulfillment.
Restriction on what we are allowed to say and write or to hear and read will hamper
our personality and its growth. It helps an individual to attain self-fulfillment.
c. For expressing belief and political attitudes - freedom of speech provides opportunity
to express one’s belief and show political attitudes. It ultimately results in the welfare
of the society and state. Thus, freedom of speech provides a mechanism by which it
would be possible to establish a reasonable balance between stability and social
change.
d. For active participation in democracy – democracy is most important feature of
today’s world. Freedom of speech is there to protect the right of all citizens to
understand political issues so that they can participate in smooth working of
democracy. That is to say, freedom of speech strengthens the capacity of an individual
in participating in decision-making.

It is apparent that the right to information was not spelt out as a separate right under Article
19. However, it is now well-settled in a catena of cases that the right to freedom of speech
and expression enshrined in Article 19(1)(a) includes the right to information.

WWW.IASBABA.COM 14
YWM5YTIyN2MzNDVk

IASBABA’S ILP 2022 – POLITY MODULE 1

In State of U.P. v. Raj Narain, (1975) 4 SCC 428, it was observed that the right to know is
derived from the concept of freedom of speech.

This was further confirmed in S.P. Gupta v. Union of India, 1981 Supp SCC 87, where it was
held that:

“The concept of an open Government is the direct emanation from the right to know which
seems to be implicit in the right of free speech and expression guaranteed under Article
19(1)(a). Therefore, disclosure of information in regard to the functioning of Government
must be the rule and secrecy an exception justified only where the strictest requirement of
public interest so demands.”

The law in this regard has been developed over the years, in Union of India v. Association for
Democratic Reforms, (2002) 5 SCC 294 and in PUCL v. Union of India, (2003) 4 SCC 399.

In consonance with its duty, Parliament enacted the Right to Information Act in 2005.

Q. What do you understand by the concept “freedom of speech and expression”? Does it
cover hate speech also? Why do the films in India stand on a slightly different plane from
other forms of expression? Discuss. (2014)

2. Right to assemble peaceably and without arms.

The right is however subject to the following restrictions. :-

• The assembly must be peaceful and harmonious;


• It must be unarmed and not threatening the safety of the people;
• Reasonable restrictions can be imposed under clause 3 of article 19.

The right to assembly embodies the very idea of a democratic government. Article 19(1)(b)
thus includes the right to hold meetings and to take out processions. However, this right is
not absolute but restrictive in nature. The assembly must be non-violent and must not breach
public peace.

Think!

• Can the state restrict a citizen’s right to protest?

3. Right to form associations or unions or co-operative societies.

The right to form associations or unions has a very wide and varied scope including all sorts
of associations viz., political parties, clubs, societies, companies, organizations,
entrepreneurships, trade unions etc.

It was held in Kulkarni’s case that the right of association pre-supposes organization. It is an
organization or permanent relationship between its members in matters of common concern.
It thus includes the right to form companies, societies, partnership, and trade union.

WWW.IASBABA.COM 15
YWM5YTIyN2MzNDVk

IASBABA’S ILP 2022 – POLITY MODULE 1

The right to form trade unions should not lead to the conclusion that trade unions have a
guaranteed right to an effective collective bargaining or to strike as a part of collective
bargaining or otherwise. The right to strike or to declare a lock-out may be controlled or
restricted by various industrial legislations such as Industrial Dispute Act or Trade Unions Act.

Right to form association does not carry the right to recognition

Right to form association does not carry the right to strike

Right to form association does not carry the right to inform rival union

4. Right to move freely throughout the territory of India.

Article 19(1)(d) of The Indian Constitution guarantees to all Citizens of India the Right "to
move freely throughout the territory of India." This Right is, however subject to reasonable
restrictions mentioned under Article 19(5). Clause (5) of Article 19 empowers the State to
impose reasonable restrictions in the interest of general public or for the protection of the
interest of any Scheduled Tribe.

According to clause (5) of Article 19 of Indian Constitution State may impose reasonable
restrictions on the Freedom of movement on two grounds:

• In the Interest of General Public


• For the Protection of Scheduled Tribes

5. Right to reside and settle in any part of the territory of India.

Article 19(1)(e) of the Indian Constitution guarantees to every citizen of India, the right "to
reside and settle in any part of the territory of India". This right is subjected reasonable
restrictions which may be imposed by the State, by law under clause (5) of Article 19, in the
interest of the general public or for the protection of the interest of any Scheduled Tribe.

Article 19(1)(d) and Article 19(1)(e) are Complementary

It is to be noted that the right to reside [under Article 19(1)(e)] and right to move [under
Article 19(1)(d)] freely throughout the Country are complementary and often go together.
Most of the Cases considered under Article 19(1)(d) are relevant to Article 19(1)(e) also. The
two rights, therefore, discussed together.

The Freedom of Movement and Residence apply only to citizens of India and not the
Foreigners. A foreigner cannot claim the right to reside and settle in the country as
guaranteed by Article 19(1)(e). The Government of India has the Power to expel foreigners
from India.

6. Right to practice any profession or to carry on any occupation, trade or business.

All citizens are given the right to practice any profession or to carry on any occupation, trade
or business. This right is very wide as it covers all the means of earning one’s livelihood.

WWW.IASBABA.COM 16
YWM5YTIyN2MzNDVk

IASBABA’S ILP 2022 – POLITY MODULE 1

The State can impose reasonable restrictions on the exercise of this right in the interest of the
general public.

The State can impose ‘reasonable’ restrictions on the enjoyment of these six rights only on
the grounds mentioned in the Article 19 itself and not on any other grounds.

ARTICLE 20: PROTECTION IN RESPECT OF CONVICTION FOR OFFENCES.

Article 20 of the Constitution is with respect to protection in respect of conviction of an


offence. It imposes limitations on the powers of the State, which it otherwise possesses under
Article 21, to enact and enforce criminal laws.

Article 20(2) is aimed at protecting an individual from being subjected to prosecution and
conviction for the same offence more than once.

Article 20(3), which protects an individual against self-incrimination, has been termed a
‘humane’ Article. It gives protection to a person accused of an offence against compulsion to
be a witness against himself. This is in consonance with the expression ‘according to
procedure established by law’, enshrined in Article 21, within the ambit of which just and fair
trials lie.

Article 20 grants protection against arbitrary and excessive punishment to an accused person,
whether citizen or foreigner or legal person like a company or a corporation.

• No ex-post-facto Legislation
• No Double Jeopardy
• No Self-incrimination

ARTICLE 21: PROTECTION OF LIFE AND PERSONAL LIBERTY.

Article 21 declares that no person shall be deprived of his life or personal liberty except
according to procedure established by law. This right is available to both citizens and non-
citizens.

Article 21 secures two rights:

• Right to life
• Right to personal liberty
The expression “Procedure established by law” is more definite phrase and this phrase finds
the place in the Japanese Constitution of 1946. It implies that life and personal liberty of a
person cannot be encroached upon arbitrarily without the proper sanction and provision of
law.

WWW.IASBABA.COM 17
YWM5YTIyN2MzNDVk

IASBABA’S ILP 2022 – POLITY MODULE 1

Though the phraseology of Article 21 starts with negative word but the word No has been
used in relation to the word deprived. The object of the fundamental right under Article 21 is
to prevent encroachment upon personal liberty and deprivation of life except according to
procedure established by law.

It clearly means that this fundamental right has been provided against state only. If, an act of
private individual amounts to encroachment upon the personal liberty or deprivation of life
of other person, such violation would not fall under the parameters set for the Article 21.

In such a case the remedy for aggrieved person would be either under Article 226 of the
constitution or under general law. But, where an act of private individual supported by the
state infringes the personal liberty or life of another person, the act will certainly come under
the ambit of Article 21. Article 21 of the Constitution deals with prevention of encroachment
upon personal liberty or deprivation of life of a person.

The Supreme Court has reaffirmed its judgment in the Menaka case in the subsequent cases.
It has declared the following rights as part of Article 21:

(a) Right to live with human dignity.

(b) Right to decent environment including pollution free water and air and protection against
hazardous industries.

® Right to livelihood.

(d) Right to privacy.

® Right to shelter.

(f) Right to health.

(g) Right to free education up to 14 years of age.

(h) Right to free legal aid.

(i) Right against solitary confinement.

(j) Right to speedy trial.

(k) Right against handcuffing.

(l) Right against inhuman treatment.

(m) Right against delayed execution.

(n) Right to travel abroad.

(o) Right against bonded labour.

(p) Right against custodial harassment.

WWW.IASBABA.COM 18
YWM5YTIyN2MzNDVk

IASBABA’S ILP 2022 – POLITY MODULE 1

(q) Right to emergency medical aid.

(r) Right to timely medical treatment in government hospital.

(s) Right not to be driven out of a state.

(t) Right to fair trial.

(u) Right of prisoner to have necessities of life.

(v) Right of women to be treated with decency and dignity.

(w) Right against public hanging.

(x) Right to hearing.

(y) Right to information.

(z) Right to reputation.

Q. Does the right to clean environment entail legal regulations on burning crackers during
Diwali? Discuss in the light of Article 21 of the Indian Constitution and Judgment(s) of the
Apex Court in this regard. (2013)

Q. Examine the scope of Fundamental Rights in the light of the latest judgement of the
Supreme Court on Right to Privacy. (2017)

Right to Reputation

• Petition filed in SC stating that the amendments to Unlawful Activities Prevention Act
(UAPA) infringed upon the fundamental right to reputation and dignity.
• Individual’s right to reputation and dignity, is considered as a facet of right to life and
personal liberty under Article 21 of the Constitution.

Key Provisions of amended UAPA Act are:

• It empowers the government to designate individuals as terrorists. Previously, only an


rganization could be designated.
• It empowers the National Investigation Agency (NIA) to seize properties, which
previously required permission from the State’s Director General of Police.
• It allows NIA officers, of the rank of Inspector or above, to investigate cases. Before
only DySP or ACP or above could do so.

The petition alleged that the amendments were considered as violative of substantive and
procedural due process of law.

WWW.IASBABA.COM 19
YWM5YTIyN2MzNDVk

IASBABA’S ILP 2022 – POLITY MODULE 1

ARTICLE 21 A – RIGHT TO EDUCATION

Article 21 A declares that the State shall provide free and compulsory education to all children
of the age of six to fourteen years in such a manner as the State may determine.

Thus, this provision makes only elementary education a Fundamental Right and not higher or
professional education.

This provision was added by the 86th Constitutional Amendment Act of 2002. This
amendment is a major milestone in the country’s aim to achieve “Education for All”.

Even before this amendment, the Constitution contained a provision for free and compulsory
education for children under Article 45 in Part IV. However, being a directive principle, it was
not enforceable by the courts. Now, there is scope for judicial intervention in this regard.

The Right of Children to Free and Compulsory Education (RTE) Act, 2009, which represents
the consequential legislation envisaged under Article 21-A, means that every child has a right
to full time elementary education of satisfactory and equitable quality in a formal school
which satisfies certain essential norms and standards.

The RTE Act provides for the:

Right of children to free and compulsory education till completion of elementary education
in a neighborhood school.

It clarifies that ‘compulsory education’ means obligation of the appropriate government to


provide free elementary education and ensure compulsory admission, attendance and
completion of elementary education to every child in the six to fourteen age group. ‘Free’
means that no child shall be liable to pay any kind of fee or charges or expenses which may
prevent him or her from pursuing and completing elementary education.

It makes provisions for a non-admitted child to be admitted to an age appropriate class.

It specifies the duties and responsibilities of appropriate Governments, local authority and
parents in providing free and compulsory education and sharing of financial and other
responsibilities between the Central and State Governments.

It lays down the norms and standards relating inter alia to Pupil Teacher Ratios (PTRs),
buildings and infrastructure, school-working days, teacher-working hours.

It provides for rational deployment of teachers by ensuring that the specified pupil teacher
ratio is maintained for each school, rather than just as an average for the State or District or
Block, thus ensuring that there is no urban-rural imbalance in teacher postings.

It also provides for prohibition of deployment of teachers for non-educational work, other
than decennial census, elections to local authority, state legislatures and parliament, and
disaster relief.
WWW.IASBABA.COM 20
YWM5YTIyN2MzNDVk

IASBABA’S ILP 2022 – POLITY MODULE 1

It provides for appointment of appropriately trained teachers, i.e. teachers with the requisite
entry and academic qualifications.

It prohibits (a) physical punishment and mental harassment; (b) screening procedures for
admission of children; (c) capitation fee; (d) private tuition by teachers and (e) running of
schools without recognition,

It provides for development of curriculum in consonance with the values enshrined in the
Constitution, and which would ensure the all-round development of the child, building on the
child’s knowledge, potentiality and talent and making the child free of fear, trauma and
anxiety through a system of child friendly and child centered learning.

ARTICLE 22: PREVENTIVE DETENTION

Article 22 is also a very important Article which has given rise to so many controversial
legislations.

The issue related to this Article is “issue of preventive detention’. Our purpose is to get the
basic idea of the “preventive Detention” and related contemporary issues. Article 22 of the
constitution of India provides that:

a) A person cannot be arrested and detained without being informed about the grounds
of such arrest.
b) This means that before a person is arrested, he/ she must be informed that he is being
arrested and reason why he / she is being arrested.
c) A person who is arrested cannot be denied to be defended by a legal practitioner of
his choice.
d) This means that the arrested person has right to hire a legal practitioner to defend
himself/ herself.
e) Every person who has been arrested would be produced before the nearest
magistrate within 24 hours.
f) The custody of the detained person cannot be beyond the said period by the authority
of magistrate.
g) The Article 22(1) and 22(2) make the above provisions. However, Article 22(3) says
that the above safeguards are not available to the following:
o If the person is at the time being an enemy alien.
o If the person is arrested under certain law made for the purpose of “Preventive
Detention”

The first condition above is justified, because when India is in war, the citizen of the enemy
country may be arrested.

WWW.IASBABA.COM 21
YWM5YTIyN2MzNDVk

IASBABA’S ILP 2022 – POLITY MODULE 1

The reasons for the introduction of such a clause were explained by Ambedkar thus:

“It has to be recognized that in the present circumstances of the country, it may be necessary
for the executive to detain a person who is tampering either with public order or with the
Defence Services of the country. In such case, I do not think that the exigency of the liberty of
the individual shall be placed above the interests of the State.”

Ambedkar, however, pointed out the safeguards provided in the Constitution to mitigate the
rigours of an apparently absolute power of preventive detention permitted under Article 22
(3).

First, every case of preventive detention must be authorized by law. It cannot be at the will
of the executive.

Secondly, no law of preventive detention shall normally authorize the detention of a person
for a longer period than three months.

Thirdly, every case of preventive detention for a period longer than three months must be
placed before an Advisory Board composed of persons qualified for appointment as Judges of
a High Court. Such cases must be placed before the Board within the three months period.

Fourthly, no person who is detained under any preventive detention law can be detained
indefinitely. There shall always be a maximum period of detention which Parliament is
required to prescribe by law.

Fifthly, in cases which are required to be placed before the Advisory Board, the procedure to
be followed by the Board shall be laid down by Parliament.

Sixthly, when a person is detained under a law of preventive detention, the detaining
authority shall communicate to him the grounds on which the order has been made. It should
also afford him the earliest opportunity of making a representation against the order.

ARTICLE 23 AND 24: RIGHT AGAINST EXPLOITATION

• Article 23 Prohibition of traffic in human beings and forced labour


• Article 23 prohibits the traffic in human beings and forced labor such as beggar.

This includes slave trade where humans were sold for the purposes of sexual exploitation,
prostitution or forced labor. It is more commonly known as the modern form of slavery as
compared to the olden times when it only meant having the slaves attend to various
household problems.

The term ‘begar’ has been used in this clause. It is a word in Indian English which means forced
labor with no compensation. When the British ruled our country, eggar system was in effect.
The British officials and Zamindars used people with poor backgrounds to carry their personal
belonging from one place to another.

WWW.IASBABA.COM 22
YWM5YTIyN2MzNDVk

IASBABA’S ILP 2022 – POLITY MODULE 1

The Zamindars, through cunning tricks deceived generations of a family into working on their
farms for free. These activities come under forced labor and were rendered illegal through
our constitution. As per the provisions enshrined in the constitution the government passed
“The Immoral Traffic (Prevention) Act 1956” and “The Bonded Labor System (Abolition) Act
1976”.

The important points are:

In case of disasters or any such emergency situation where the government needs additional
help from civilian workforce, even then it can’t have them work without paying any
remuneration. It still has to pay those workers the minimum wage set at that time.

We all know that the prisoners get paid for the work they do in jail. There are two cases here.
It is not a violation of Article 23 if the prisoners who are sentenced to rigorous punishment
are not paid for their work. However, any person who is under preventive detention, normal
sentence or those under trials must be paid reasonable wages if they want to work.

Now, let’s understand another related term, Bonded Labour.

It is also called forced labor. As clear the name is, it doesn’t simply mean forced manual work,
but also the work done due to economical compulsions. For example, if a person is forced to
take up work that pays less than the predefined minimum wage, it is also a violation.

ARTICLE 24: PREVENTION OF CHILD LABOUR

Article 24 mandates that – ‘No child below age of 14 years shall be employed to work in any
factory or mine or engaged in any other hazardous employment’.

The Commissions for Protection of Child Rights Act, 2005 was enacted to provide for the
establishment of a National Commission and State Commissions for Protection of Child Rights
and Children’s Courts for providing speedy trial of offences against children or of violation of
child rights.

In 2006, the government banned the employment of children as domestic servants or workers
in business establishments like hotels, dhabas, restaurants, shops, factories, resorts, spas,
tea-shops and so on. It warned that anyone employing children below 14 years of age would
be liable for prosecution and penal action.

Indian government had promulgated a legislation Child and Adolescent Labour (Prohibition
and Regulation) Act, 1986 ("CL Act") to regulate the child labour practices in India.

The central legislature has made substantial changes in the provisions of the CL Act in the year
2016 and the said amendments have been made effective from July 30, 2016.

Pursuant to the said amendment the name of the CL Act has been changed to 'Child and
Adolescent Labour (Prohibition and Regulation) Act, 1986'. A complete prohibition has been

WWW.IASBABA.COM 23
YWM5YTIyN2MzNDVk

IASBABA’S ILP 2022 – POLITY MODULE 1

imposed on employment of child labour (i.e. a person below the age of 14 years) in any
establishment whether hazardous or not.

Features

• A child is permitted to work only to help family, in family enterprise or as child artist
after school hours or during vacations.
• The amendment has introduced the concept of adolescent labour for the first time.
• An adolescent has been defined as a person between the ages of 14-18 years.
• The amendment permits employment of adolescent labour except in hazardous
processes or occupation.
• The offences under the Act have now been made compoundable and cognizable
notwithstanding the provisions of the Criminal Procedure Code.
• The CL Act provides for rehabilitation of children and adolescent who have been
victims under the provisions of the CL Act.
• It provides for setting up of the Child and Adolescent Labour Rehabilitation Fund in
which all the amounts of penalty have to be realised.
• Liability has been affixed upon the parents and guardian of the affected child/children
separately from the employers.
• The Act provides for increased penalty and imprisonment which shall not be less than
6 months and may extend upto 2 years and fine which may vary between Rs. 20, 000
to Rs. 50,000.
• Previously, the violations under the CL Act were punishable with imprisonment of not
less than three months which could extend to one year or/and with fine of ten
thousand rupees which could extend to twenty thousand rupees.

Critical Assessment

The new amendments put a complete prohibition on employment of children, but at the
same time it allows them to be employed in family enterprises/businesses.

Considering that majority of child labor activities happen in economically weaker section of
the society which is highly unregulated, no proper mechanism has been provided to keep
the same in check with the new amendments.

According to the UNICEF India, permitting children to work in their family enterprises would
lead to more children working in unregulated conditions. The Section 3 which has this
provision does not even specify the hours of work. It simply states that children can help after
school hours or during vacations. It may restrict the children especially the poor children
belonging to low caste to traditional caste-based occupations for generations. Also, it may be
difficult to determine whether an enterprise is owned by a family or some person has
employed the whole family to run the enterprise.

WWW.IASBABA.COM 24
YWM5YTIyN2MzNDVk

IASBABA’S ILP 2022 – POLITY MODULE 1

Further, the list of hazardous industries has been drastically decreased, this may allow the
employers in industries like chemical mixing units, cotton farms, battery recycling units, and
brick kilns etc. (which are actually hazardous) to employ adolescent labour, which they may
even get at a much cheaper price.

It is therefore more important now for the government to keep a check on the working
conditions for adolescent labour as well as the working conditions for children in family run
businesses. This would require more personnel deployment which currently is in shortage.

THINK!

• Critically analyse recent amendments made to India’s child labour law.


• Do you think the existing legal and institutional support is adequate to end the menace
of child labour in India? Critically comment.

ARTICLE 25 - 28: RIGHT TO RELIGIOUS FREEDOM

ARTICLE 25: FREEDOM OF CONSCIENCE AND FREE PROFESSION, PRACTICE AND


PROPAGATION OF RELIGION

Article 25 says that all persons are equally entitled to freedom of conscience and the right to
freely profess, practice and propagate religion. The implications of these are:

Freedom of conscience: Inner freedom of an individual to mould his relation with God or
Creatures in whatever way he desires.

Right to profess: Declaration of one's religious beliefs and faith openly and freely.

Right to practice: Performance of religious worship, rituals, ceremonies and exhibition of


beliefs and ideas.

Right to propagate: Transmission and dissemination of one's religious beliefs to others or


exposition of the tenets of one's religion. But it does not include a right to convert another
person to one's own religion. Forcible conversions impinge on the ‘freedom of conscience'
guaranteed to all the persons alike.

Article 25 covers not only Religious beliefs (doctrines) but also religious practices (rituals).

These Rights are available to all person— citizens as well as non-citizens.

Further, the state can impose reasonable restrictions on these Rights on the following
grounds:

• Public Order
• Morality
• Health

WWW.IASBABA.COM 25
YWM5YTIyN2MzNDVk

IASBABA’S ILP 2022 – POLITY MODULE 1

• Other provisions relating to Fundamental Rights

Further Article 25 also permits the State to:

1. Regulate or restrict any economic, financial, political or other secular activity


associated with religious practice; (It means the state will not interfere in religious
matters but it can interfere the economic, financial, political and other matters of any
religion or religious practice)
2. Provide for social welfare and reform or throw open Hindu religious institutions of
public character to all classes and sections of Hindus. ( In Hindu Religion there are
caste system and a lot of discrimination on the grounds of Castes, so the article 25
authorizes the state to open the Hindu Religious Institutions for all Hindus)

Article 25 also contains two explanations:

1. Wearing and carrying of kripans is to include in the profession of the Sikh religion
2. the Hindus, in this context, include, Sikhs, Jains and Buddhists.

RIGHT OF BEING NOT BELONGING TO ANY RELIGION – ARTICLE 25


• The Punjab and Haryana high court has held that if a person has chosen a path of an
Atheist and not to believe in any caste, class, there would be no requirement in law
for him to be issued a certificate to such effect.
• The court ruled that it cannot issue directions to authorities to issue the certificate of
being an “Atheist” or a person of no caste, no religion, and no God to any person.
• The Court was hearing a plea from 33-year-old Ravi Kumar Atheist who had moved
the court after the Fatehabad district authorities in April withdrew the ‘No God
Certificate’ they had issued only a week earlier, citing “wrong interpretation of civil
court orders”.

Following were the High Court observations

• The freedom of conscience under Article 25 of the Constitution encompasses in itself


a freedom to an individual to take a view that he does not belong to any religion.
• It includes a right of an individual to claim that he is an ‘Atheist’.
• Just as a freedom of conscience confers a fundamental right to a citizen to entertain a
particular religious belief, it equally confers a right on any other individual/citizen to
express an opinion that he does not belong to any religion.

WWW.IASBABA.COM 26
YWM5YTIyN2MzNDVk

IASBABA’S ILP 2022 – POLITY MODULE 1

ARTICLE 26: FREEDOM TO MANAGE RELIGIOUS AFFAIRS


According to Article 26, every religious denomination or any of its section shall have the
following rights:

• Right to establish and maintain institutions for religious and charitable purposes;
• Right to manage its own affairs in matters of religion;
• Right to own and acquire movable and immovable property; and
• Right to administer such property in accordance with law.

Religious Denomination: Meaning and Scope

It has now been established through various judgments that three conditions must be
satisfied in order to qualify as a religious denomination under Article 26. These are:

a) It must be a collection of individuals who have a system of beliefs which they regard
as conducive to their spiritual well-being, i.e., common Faith
b) Common Organisation
c) Designated by distinctive name

Therefore, members belonging to different religions, satisfying the three tests, would be a
denomination within the meaning of Article 26. The expression ‘denomination’ can also be
used for members forming sects or sub-sects of a religion designated by a distinctive Name.

It is pertinent to note that, unlike Article 30, benefit of Article 26 is not confined to ‘minority’
groups only. Sikhs, not in minority in Punjab, constitute a ‘religious denomination’13 and can
thus, claim the benefit of Article 26.

ARTICLE 27: FREEDOM FROM TAXATION FOR PROMOTION OF A RELIGION


Article 27 lays down that no person shall be compelled to pay any taxes for the promotion or
maintenance of any particular religion or religious denomination.

In other words, the State should not spend the public money collected by way of tax for the
promotion or maintenance of any particular religion.

This provision prohibits the State from favoring, patronizing and supporting one religion over
the other.

This means that the taxes can be used for the promotion or maintenance of all religions.

The provision prohibits only levy of a tax not a fee. This is because the purpose of a fee is to
control the secular administration of religious institutions and not to promote or maintain
religion. Thus, a fee can be levied on pilgrims to provide them some special service or safety
measures.

Similarly, a fee can be levied on religious endowments for meeting the regulation
expenditure.

WWW.IASBABA.COM 27
YWM5YTIyN2MzNDVk

IASBABA’S ILP 2022 – POLITY MODULE 1

ARTICLE 28: FREEDOM FROM ATTENDING RELIGIOUS INSTRUCTION


Under Article 28, no religious instruction shall be provided in any educational institution
wholly maintained out of State funds.

However, this provision shall not apply to an educational institution administered by the State
but established under any endowment or trust, requiring imparting of religious instruction in
such institution.

According to Article 28 (2), the above provision shall not apply to an educational institution
administered by the State but established under any endowment or trust, requiring imparting
of religious instruction in such institutions.

According to Article 28 (3), No person attending any educational institution recognised by


the State or receiving aid out of State funds shall be required to attend any religious
instruction or worship in that institution without his consent. In case of a minor the consent
of his guardian is needed.

Thus, article 28 distinguishes between four types of educational institutions:

1. Institutions wholly maintained by the State


2. Institutions administered by the State but establish under any endowment or trust.
3. Institutions recognised by the State.
4. Institutions receiving aid from the State.

In (type-1 institutions) the Institutions wholly maintained by the State, religious instructions
is completely prohibited.

In (type-2 institutions) the Institutions administered by the State but establish under any
endowment or trust, religious instructions is permitted.

In (type-3 and 4 institutions) Instintutions recognized by the State and receiving aid from the
State, religious instruction is permitted on a voluntary basis.

ARTICLE 29 AND 30: CULTURAL AND EDUCATIONAL RIGHTS

ARTICLE 29: PROTECTION OF INTERESTS OF MINORITIES


Article 29 protects the interests of the minorities by making a provision that any citizen /
section of citizens having a distinct language, script or culture have the right to conserve the
same.

Article 29 mandates that no discrimination would be done on the ground of religion, race,
caste, language or any of them.

WWW.IASBABA.COM 28
YWM5YTIyN2MzNDVk

IASBABA’S ILP 2022 – POLITY MODULE 1

Principles of Article 29

The Bombay High Court held that it embodied two important principles under Article 29:

“One is the right of the citizen to select any educational institution maintained by the State
and receiving aid out of State funds. The State cannot tell a citizen, ‘you shall go to this school
which I maintain and not to the other’.

Features of clauses under Article 29

It protects the language, script or culture of sections of the citizens.

It guarantees the fundamental right of an individual citizen. The right to get admission into
any educational institution of the kind mentioned.

It is a right which an individual citizen has as a citizen and not as a member of any community
or class of citizens.

On the other hand, if he has the academic qualifications but is refused admission only on
grounds of religion, race, caste, language or any of them, then there is a clear breach of his
fundamental right.”

But the Court rejected this argument on the ground that this was a Directive Principle a non-
justiciable right and it could not override a Fundamental Right which was justiciable. It was
the duty of the Court to enforce a Fundamental Right.

With the passing of the Forty-Second Amendment of the Constitution this argument of the
Court has lost much of its force. According to the Amendment where there is a conflict
between a Fundamental Right and Directive Principle, Parliament may by law give precedence
to the Directive Principle.

Article 29, enunciates the Fundamental Right of any section of citizens residing anywhere in
India to conserve its distinct language, script or culture. No citizen can be denied admission
to any educational institution maintained or aided by the state on grounds of language or
religion.

ARTICLE 30: RIGHT OF MINORITIES TO ESTABLISH AND ADMINISTER EDUCATIONAL


INSTITUTIONS
Article 30 mandates that all minorities, whether based on religion or language, shall have the
right to establish and administer educational institutions of their choice.

Article 30 is called a Charter of Education Rights.

Madarasa are administrated by the Article 30.

WWW.IASBABA.COM 29
YWM5YTIyN2MzNDVk

IASBABA’S ILP 2022 – POLITY MODULE 1

Article 30 provides an absolute right to the minorities that they can establish their own
linguistic and religious institutions and at the same time can also claim for grant-in-aid
without any discrimination.

The term ‘minority’ has not been defined anywhere in the Constitution.

The right under Article 30 also includes the right of a minority to impart education to its
children in its own language.

Minority educational institutions are of three types:

a) institutions that seek recognition as well as aid from the State;


b) institutions that seek only recognition from the State and not aid; and
c) institutions that neither seek recognition nor aid from the State.

Article 31: Repealed

ARTICLE 32: RIGHT TO CONSTITUTIONAL REMEDIES


Article 32 was called “the very soul of the constitution and the very heart of it” by Dr. B R
Ambedkar.

Mere declaration of the fundamental right is meaningless until and unless there is an effective
machinery for enforcement of the fundamental rights.

So, a right without a remedy is a worthless declaration. The framers of our constitution
adopted the special provisions in the article 32 which provide remedies to the violated
fundamental rights of a citizen.

WWW.IASBABA.COM 30
YWM5YTIyN2MzNDVk

IASBABA’S ILP 2022 – POLITY MODULE 1

WWW.IASBABA.COM 31
YWM5YTIyN2MzNDVk

IASBABA’S ILP 2022 – POLITY MODULE 1

Article 32 (1) says: The right to move the Supreme Court by appropriate proceedings for the
enforcement of the rights conferred by this Part is guaranteed.

Article 32 (2) says: The Supreme Court shall have power to issue directions or orders or writs,
including writs in the nature of habeas corpus, mandamus, prohibition, quo warranto and
certiorari, whichever may be appropriate, for the enforcement of any of the rights conferred
by this Part.

Why Dr. Ambedkar considered the right to constitutional remedies as “heart and soul of
the constitution”?

Because this right gives a citizen the right to approach a High Court or the Supreme Court to
get any of the fundamental rights restored in case of their violation.

The Supreme Court and the High Courts can issue orders and give directives to the
government for the enforcement of rights.

What are the writs issued by the court?

The courts can issue various special orders known as writs.

Habeas corpus: Means that the court orders that the arrested person should be presented
before it. It can also order to set free an arrested person if the manner or grounds of arrest
are not lawful or satisfactory.

Mandamus: Issued when the court finds that a particular office holder is not doing legal duty
and thereby is infringing on the right of an individual.

Prohibition: Issued by a higher court (High Court or Supreme Court) when a lower court has
considered a case going beyond its jurisdiction.

Quo Warranto: If the court finds that a person is holding office but is not entitled to hold that
office, it issues the writ of quo warranto and restricts that person from acting as an office
holder.

Certiorari: Under this writ, the court orders a lower court or another authority to transfer a
matter pending before it to the higher authority or court.

WWW.IASBABA.COM 32
YWM5YTIyN2MzNDVk

IASBABA’S ILP 2022 – POLITY MODULE 1

ARMED FORCES AND FUNDAMENTAL RIGHTS


Article 33 & 34 empower the Parliament to restrict, modify or abrogate the fundamental
rights to the members of armed forces, para-military forces, police forces, members of
intelligence agencies or similar services.

This is required to make the proper discharge of their duties which are sensitive and urgent
in nature. This power is available only with parliament and not state legislatures. Further, such
an act cannot be challenged in a court on ground of its being of violative of fundamental
rights.

Further, court martial (tribunals under the military law) have been exempted from the writ
jurisdiction of the Supreme Court and the high courts under article 33.

Using these powers, the parliament enacted various laws such as Army Act (1950), Navy Act
(1950), Air Force Act (1950), Police Forces (Restriction of Rights) Act, 1966 etc.

Parliament has also the power to indemnify any person or the acts done by such person in
the service of the State, if such acts are done by him during the enforcement of martial law in
any area within the territory of India.

Martial Law and Fundamental Rights

Article 34 provides for the restrictions on fundamental rights while martial law is in force in
any area within the territory of India.

It empowers the Parliament to indemnify any government servant or any other person for
any act done by him in connection with the maintenance or restoration of order in any area
where martial law was in force.

The Act of Indemnity made by the Parliament cannot be challenged in any court on the ground
of contravention of any of the fundamental rights.

The expression ‘martial law’ has not been defined anywhere in the Constitution.

The Supreme Court held that the declaration of martial law does not ipso facto result in the
suspension of the writ of habeas corpus.

EFFECTING CERTAIN FUNDAMENTAL RIGHTS


Article 35 lays down that the power to make laws, to give effect to certain specified
fundamental rights shall vest only in the Parliament and not in the state legislatures.

This provision ensures that there is uniformity throughout India with regard to the nature of
those fundamental rights and punishment for their infringement.

WWW.IASBABA.COM 33
YWM5YTIyN2MzNDVk

IASBABA’S ILP 2022 – POLITY MODULE 1

CRITICISM OF FUNDAMENTAL RIG HTS


The FRs enshrined in Part III of the Constitution have met with a wide range and varied
criticism.

Excessive Limitations

They are subjected to innumerable exceptions, restrictions, qualifications and explanations.


Hence, the critics remarked that the Constitution grants Fundamental Rights with one hand
and takes them away with the other.

No Social and Economic Rights

The list is not comprehensive as it mainly consists of political rights. It makes no provision for
important social and economic rights like right to social security, right to work, right to
employment, right to rest and leisure and so on.

No Clarity

It is alleged that the Constitution was made by the lawyers for the lawyers. Sir Ivor Jennings
called the Constitution of India a ‘paradise for lawyers’. The various phrases and words used
in the chapter like ‘public order’, ‘minorities’, ‘reasonable restriction’, ‘public interest’ and so
on are not clearly defined.

No Permanency

They are not sacrosanct or immutable as the Parliament can curtail or abolish them, as for
example, the abolition of the fundamental right to property in 1978. Hence, they can become
a play tool in the hands of politicians having majority support in the Parliament.

The judicially innovated ‘doctrine of basic structure’ is the only limitation on the authority of
Parliament to curtail or abolish the fundamental right.

Suspension during Emergency

The suspension of their enforcement during the operation of National Emergency (except
Articles 20 and 21) is another blot on the efficacy of these rights.

Expensive Remedy

The judicial process is too expensive and hinders the common man from getting his rights
enforced through the courts. Hence, the critics say that the rights benefit mainly the rich
section of the Indian Society.

Preventive Detention

No democratic country in the world has made preventive detention as an integral part of their
Constitutions as has been made in India. It confers arbitrary powers on the State and negates
individual liberty.

WWW.IASBABA.COM 34
YWM5YTIyN2MzNDVk

IASBABA’S ILP 2022 – POLITY MODULE 1

No Consistent Philosophy

Sir Ivor Jennings expressed this view when he said that the Fundamental Rights proclaimed
by the Indian Constitution are based on no consistent philosophy.25 The critics say that this
creates difficulty for the Supreme Court and the high courts in interpreting the fundamental
rights.

SIGNIFICANCE OF FUNDAMENTAL RIGHTS


In spite of the above criticism and shortcomings, the Fundamental Rights are significant in the
following respects:

• They constitute the bedrock of democratic system in the country.


• They provide necessary conditions for the material and moral protection of man.
• They serve as a formidable bulwark of individual liberty.
• They facilitate the establishment of rule of law in the country.
• They protect the interests of minorities and weaker sections of society.
• They strengthen the secular fabric of the Indian State.
• They check the absoluteness of the authority of the government.
• They lay down the foundation stone of social equality and social justice.
• They ensure the dignity and respect of individuals.
• They facilitate the participation of people in the political and administrative process.

WWW.IASBABA.COM 35
YWM5YTIyN2MzNDVk

IASBABA’S ILP 2022 – POLITY MODULE 1

DIRECTIVE PRINCIPLES OF STATE POLICY (DPSP)


Directive Principle of State Policy provides guidelines to Central and State government in
India, to be kept in mind while framing laws and policies and are mentioned in part 4 of the
constitution.

The provisions contained in this Part cannot be enforced by any court, but these principles
are fundamental in the governance of the country and it shall be the duty of the State to
apply these principles in making laws.

While most of the Fundamental Rights are negative obligations on the state, DPSP are positive
obligations on the state, though not enforceable in a court of law.

The framers of the Constitution borrowed this idea from the Irish Constitution of 1937, which
had copied it from the Spanish Constitution.

Dr B R Ambedkar described these principles as ‘novel features’ of the Indian Constitution.

DPSP + FR Conscience of Indian Constitution

DPSP + Fundamental Rights soul of the Constitution

Q. According to the Constitution of India, which of the following are fundamental for the
governance of the country? (2013)

a) Fundamental Rights
b) Fundamental Duties
c) Directive Principles of State Policy
d) Fundamental Rights and Fundamental Duties

FEATURES OF THE DIRECTIVE PRINCIPLES


• Basic aim of DPSPs is to set up social and economic goals before the law makers
• To bring socio-economic change in the country
• To fulfill the basic needs of the common man
• To reshape the structure of Indian society in direction of greater socio-economic
equality

DPSPs are fundamentals in governance of the country and shall be considered dutifully by the
state while making laws, but DPSPs are not enforceable in court of law.

• If state fails to fulfill these obligations, one cannot go to court of law


• DPSPs only provides a yardstick for measuring success or failure of the government

WWW.IASBABA.COM 36
YWM5YTIyN2MzNDVk

IASBABA’S ILP 2022 – POLITY MODULE 1

Articles 36 to 51 deal with the provisions of the Directive Principles and are broadly classified
into

1. Socialist principles
2. Gandhian principles
3. Liberal intellectual principles

WWW.IASBABA.COM 37
YWM5YTIyN2MzNDVk

IASBABA’S ILP 2022 – POLITY MODULE 1

SOCIALIST PRINCIPLES
1. To secure a social order for the promotion of welfare of the people.
2. To strive to minimize inequalities of income i.e. operation of the economic system
does not result in the concentration of wealth and means of production to the
common detriment;
3. ownership and control of the material resources of the community are so distributed
as best to sub serve the common good;
4. Equal justice and free legal aid.
5. Ownership and control of material resources of the community shall be so distributed
so as to sub serve the common good.
6. Equal pay for equal work.
7. Health and strength of workers, and the tender age of children must not be abused.
8. Right to work, to education and to public assistance in certain cases.
9. Provision of just and humane conditions for work and maternity relief.
10. Participation of workers in the management of the industries.
11. Duty of the State to raise the level of nutrition and the standard of living and to
improve public health.
12. Children are given opportunities and facilities to develop in a healthy manner and in
conditions of freedom and dignity and that childhood and youth are protected against
exploitation and against moral and material abandonment.

LIBERAL PRINCIPLES
1. Uniform Civil Code for the citizens.
2. Provide free and compulsory education for children below 14 years.
3. Separation of Judiciary from Executive.
4. To promote international peace and amity.
5. Protection of monuments and places and objects of national importance
6. Protection and improvement of environment and safeguarding of forests and wild life.

THE GANDHIAN PRINCIPLES


1. Organization of Village Panchayats and to promote cottage industry.
2. Promotion of educational and economic interests of the SCs, the STs and the other
weaker sections of the society.
3. To bring about the prohibition of intoxicating drinks and drugs that are injurious to
health.
4. Organization of agriculture and animal husbandry on modern and scientific lines to
prohibit the slaughter of cows, calves and other milch and draught animals.

ARTICLE 36: DEFINITION


In this Part, unless the context otherwise requires, “the State” has the same meaning as in
Part III.

WWW.IASBABA.COM 38
YWM5YTIyN2MzNDVk

IASBABA’S ILP 2022 – POLITY MODULE 1

ARTICLE 37: APPLICATION OF THE PRINCIPLES CONTAINED IN THIS PART


The provisions contained in this Part shall not be enforced by any court, but the principles
therein laid down are nevertheless fundamental in the governance of the country and it shall
be the duty of the State to apply these principles in making laws.

ARTICLE 38: STATE TO SECURE A SOCIAL ORDER FOR THE PROMOTION OF WELFARE OF
THE PEOPLE
1) The State shall strive to promote the welfare of the people by securing and protecting
as effectively as it may a social order in which justice, social, economic and political,
shall inform all the institutions of the national life.
2) The State shall, in particular, strive to minimise the inequalities in income, and
endeavour to eliminate inequalities in status, facilities and opportunities, not only
amongst individuals but also amongst groups of people residing in different areas or
engaged in different vocations.

Q. ‘Economic Justice’ as one of the objectives of the Indian Constitution has been provided
in (2013)

a) The Preamble and the Fundamental Rights.


b) The Preamble and the Directive Principles of State Policy.
c) The Fundamental Rights and the Directive Principles of State Policy.
d) None of the above

ARTICLE 39: CERTAIN PRINCIPLES OF POLICY TO BE FOLLOWED BY THE STATE


a) The State shall, in particular, direct its policy towards securing –
b) that the citizen, men and women equally, have the right to an adequate means of
livelihood;
c) that the ownership and control of the material resources of the community are so
distributed as best to subserve the common good;
d) that the operation of the economic system does not result in the concentration of
wealth and means of production to the common detriment;
e) that there is equal pay for equal work for both men and women;
f) that the health and strength of workers, men and women, and the tender age of
children are not abused and that citizens are not forced by economic necessity to
enter avocations unsuited to their age or strength;
g) that children are given opportunities and facilities to develop in a healthy manner and
in conditions of freedom and dignity and that childhood and youth are protected
against exploitation and against moral and material abandonment.

ARTICLE 39A: EQUAL JUSTICE AND FREE LEGAL AID

WWW.IASBABA.COM 39
YWM5YTIyN2MzNDVk

IASBABA’S ILP 2022 – POLITY MODULE 1

The State shall secure that the operation of the legal system promotes justice, on a basis of
equal opportunity, and shall, in particular, provide free legal aid, by suitable legislation or
schemes or in any other way, to ensure that opportunities for securing justice are not denied
to any citizen by reason of economic or other disabilities.

ARTICLE 40: ORGANISATION OF VILLAGE PANCHAYATS


The State shall take steps to organise village panchayats and endow them with such powers
and authority as may be necessary to enable them to function as units of self-government.

ARTICLE 41: RIGHT TO WORK, TO EDUCATION AND TO PUBLIC ASSISTANCE IN CERTAIN


CASES
The State shall, within the limits of its economic capacity and development, make effective
provision for securing the right to work, to education and to public assistance in cases of
unemployment, old age, sickness and disablement, and in other cases of undeserved want.

ARTICLE 42: PROVISION FOR JUST AND HUMANE CONDITIONS OF WORK AND MATERNITY
RELIEF
The State shall make provision for securing just and humane conditions of work and for
maternity relief.

ARTICLE 43: LIVING WAGE, ETC., FOR WORKERS


The State shall endeavor to secure, by suitable legislation or economic organisation or in any
other way, to all workers agricultural, industrial or otherwise, work, a living wage, conditions
of work ensuring a decent standard of life and full enjoyment of leisure and social and cultural
opportunities and, in particular, the State shall endeavour to promote cottage industries on
an individual or co-operative basis in rural areas.

ARTICLE 43A: PARTICIPATION OF WORKERS IN MANAGEMENT OF INDUSTRIES


The State shall take steps, by suitable legislation or in any other way, to secure the
participation of workers in the management of undertakings, establishments or other
organisation engaged in any industry.

ARTICLE 44: UNIFORM CIVIL CODE FOR THE CITIZEN


The State shall endeavour to secure for the citizens a uniform civil code throughout the
territory of India.

ARTICLE 45: PROVISION FOR FREE AND COMPULSORY EDUCATION FOR CHILDREN
Substitution of new article for article 45.- For article 45 of the Constitution, the following
article shall be substituted, namely: -

Provision for early childhood care and education to children below the age of six years.

The State shall endeavor to provide early childhood care and education for all children until
they complete the age of six years."

WWW.IASBABA.COM 40
YWM5YTIyN2MzNDVk

IASBABA’S ILP 2022 – POLITY MODULE 1

ARTICLE 46: PROMOTION OF EDUCATIONAL AND ECONOMIC INTERESTS OF SCHEDULED


CASTES, SCHEDULED TRIBES AND OTHER WEAKER SECTIONS
The State shall promote with special care the educational and economic interests of the
weaker sections of the people, and in particular, of the Scheduled Castes and the Scheduled
Tribes, and shall protect them from social injustice and all forms of exploitation.

ARTICLE 47: DUTY OF THE STATE TO RAISE THE LEVEL OF NUTRITION AND THE STANDARD
OF LIVING AND TO IMPROVE PUBLIC HEALTH
The State shall regard the raising of the level of nutrition and the standard of living of its
people and the improvement of public health as among its primary duties and, in particular,
the State shall endeavour to bring about prohibition of the consumption except for medicinal
purpose of intoxicating drinks and of drugs which are injurious to health.

ARTICLE 48: ORGANIZATION OF AGRICULTURE AND ANIMAL HUSBANDRY


The State shall endeavour to organize agriculture and animal husbandry on modern and
scientific lines and shall, in particular, take steps for preserving and improving the breeds, and
prohibiting the slaughter, of cows and calves and other milch and draught cattle.

ARTICLE 48A: PROTECTION AND IMPROVEMENT OF ENVIRONMENT AND SAFEGUARDING


OF FORESTS AND WILD LIFE
The State shall endeavour to protect and improve the environment and to safeguard the
forests and wild life of the country.

ARTICLE 49: PROTECTION OF MONUMENTS AND PLACES AND OBJECTS OF NATIONAL


IMPORTANCE
It shall be the obligation of the State to protect every monument or place or object of artistic
or historic interest, declared by or under law made by Parliament to be of national
importance, from spoliation, disfigurement, destruction, removal, disposal or export, as the
case may be.

ARTICLE 50: SEPARATION OF JUDICIARY FROM EXECUTIVE


The State shall take steps to separate the judiciary from the executive in the public services
of the State.

ARTICLE 51: PROMOTION OF INTERNATIONAL PEACE AND SECURITY


a) The State shall endeavour to –
b) promote international peace and security;
c) maintain just and honourable relations between nations;
d) foster respect for international law and treaty obligations in the dealings of organised
people with one another; and
e) encourage settlement of international disputes by arbitration.

WWW.IASBABA.COM 41
YWM5YTIyN2MzNDVk

IASBABA’S ILP 2022 – POLITY MODULE 1

Q. Consider the following provisions under the Directive Principles of State Policy as
enshrined in the Constitution of India. (2012)

1. Securing for citizens of India a uniform civil code.


2. Organizing village Panchayats.
3. Promoting cottage industries in rural areas.
4. Securing for all the workers reasonable leisure and cultural opportunities.
Which of the above are the Gandhian Principles that are reflected in the Directive
Principles of State Policy?

a) 1, 2 and 4 only
b) 2 and 3 only
c) 1, 3 and 4 only
d) 1, 2 ,3 and 4

Q. Which of the following provisions of the Constitution of India have a bearing on


Education?

1. Directive Principles of State Policy


2. Rural and Urban Local Bodies
3. Fifth Schedule
4. Sixth Schedule
5. Seventh Schedule
Select the correct answer using the codes given below:

a) 1 and 2 only
b) 3, 4 and 5 only
c) 1, 2 and 5 only
d) 1, 2, 3, 4 and 5

Q. The 42nd Amendment Act of 1976 added four new Directive Principles to the original
list. They are

1) To secure opportunities for healthy development of children (Article 39).


2) To promote equal justice and to provide free legal aid to the poor (Article 39 A).
3) To take steps to secure the participation of workers in the management of industries
(Article 43 A).
4) To protect and improve the environment and to safeguard forests and wild life
(Article 48 A).

WWW.IASBABA.COM 42
YWM5YTIyN2MzNDVk

IASBABA’S ILP 2022 – POLITY MODULE 1

Q. Which principle among the ‘following was added to the Directive Principles of State
Policy by the 42nd Amendment to the Constitution? (2017)

a) Equal pay for equal work for both men and women
b) Participation of workers in the management of industries
c) Right to work, education and public assistance
d) Securing living wage and human conditions of work to workers

The 44th Amendment Act of 1978 added one more Directive Principle, which requires the
State to minimize inequalities in income, status, facilities and opportunities (Article 38).

The 86th Amendment Act of 2002 changed the subject-matter of Article 45 and made
elementary education a fundamental right under Article 21 A.

The 97th Amendment Act of 2011 added a new Directive Principle relating to co-operative
societies.

It requires the state to promote voluntary formation, autonomous functioning, democratic


control and professional management of co-operative societies (Article 43B).

DIRECTIVES IN OTHER PARTS OF THE CONSTITUTION (EXCEPT PART IV)

Article 350 A: It enjoins every State and every local authority within the State to provide
adequate facilities for the instructions in the mother tongue at the primary stage to children
of linguistic minority areas.

Article 351: It enjoins the Union to promote the spread of Hindi Language so that it may serve
as a medium of expression of all the elements of the composite culture of India.

Article 335: It says that the claims of SC/ST shall be taken into consideration, consistently with
the maintenance of efficiency of administration, in the making of appointments to services
and posts in connection with affairs of the Union or of a State.

Under the implementation of DPSP, Zamindari, Jaghirdari and Inamdari systems were
abolished and actual tillers of the soil were made owner of the land.

WWW.IASBABA.COM 43
YWM5YTIyN2MzNDVk

IASBABA’S ILP 2022 – POLITY MODULE 1

CRITICISM TO DPSP
Many critics have been very vocal in criticizing the existence of unenforceable pious
declarations (Directive Principles) in the Constitution of India.

K.T. Shah compares DPSP to a cheque payable by the bank at its convenience.

MAIN POINTS OF CRITICISM:


Lack of Legal Force:

The critics hold that as unenforceable directives, these principles do not carry any weight.
Their violation or non-realization cannot be challenged in any court.

Mere Declarations:

The Directive Principles are mere declaration of intentions or instructions which are to be
observed and secured by the State at will. The Constitution neither makes them justiciable
nor fixes the time- limit within which these are to be secured.

Unsystematic Enumeration and No Classification:

Another point of criticism against the Directive Principles has been that these have been
neither systematically stated nor properly classified. These appear to be a collection of some
pious declarations which have only a moral value.

Lack of Clarity:

Several Directives lack clarity. Several principles have been repeated at different places. The
Directive to promote international peace and friendly cooperation among all the nations is a
laudable declaration. But the real issue is how to secure it? No clear guideline has been given
for this purpose.

Reactionary in Nature:

Many critics hold that written during 1947-49, several of the Directives appear to be
reactionary in contemporary times. The party in power at a particular time can use some of
the directives for political and selfish ends. Moreover, enumeration of these principles
involves an attempt to unduly bind the present with the past.

Impracticability of some of the Principles:

Part IV includes some directives which cannot be realized in actual practice. The ideal is to
introduce prohibition, but this ideal cannot be really and effectively realized. The states which
introduced prohibition had to later on scrap it.

WWW.IASBABA.COM 44
YWM5YTIyN2MzNDVk

IASBABA’S ILP 2022 – POLITY MODULE 1

Obsolete Philosophical Foundations:

Most of the Directive Principles incorporated in this part of the Constitution are based on age
old and foreign philosophical foundations (Fabian Socialism). The philosophy of Fabian
Socialism has lost much of its relevance in contemporary times.

Superfluous:

Many critics hold that the Directive Principles merely restate the objectives and goals clearly
stated in the Preamble of the Constitution. Their description in Part IV has made things more
complex and complicated.

Mere Promises:

Directive principles are designed to serve as pious promises for creating an impression about
a just exercise of the power of the State. Their aim is to secure support through promise-
making and not action. On the basis of these arguments the critics severely riticize the
existence and scope of Part IV of the Constitution.

SIGNIFICANCE OF DIRECTIVE PRINCIPLES

Directive Principles are backed by Public Opinion:

It is true that Directive Principles are non-justiciable. These are not backed by legal sanctions.
However, these are backed by public opinion, which is in reality the real sanction behind every
law.

Provide for a Welfare State:

The Directive Principles clearly lay down the philosophical foundations of a welfare polity.
These make it a responsibility of the State to secure it through welfare legislation. These also
provide that a welfare state stands for securing of Justice—social, economic and political for
all the people.

Importance as Moral Ideals:

Directive Principles are indeed of the nature of moral ideals. They constitute a moral code for
the State. This does not reduce their value. Through these the founding fathers placed before
the nation the goals and ideals which are to be achieved through future legislation.

State is a human social institution. Government is always made and managed by the people.
Just as people have a moral code which guides their behavior in society, likewise there is every
justification for the existence of a moral code for the men who form and run the government
of the state.

WWW.IASBABA.COM 45
YWM5YTIyN2MzNDVk

IASBABA’S ILP 2022 – POLITY MODULE 1

Directives Constitute a Guide for the State:

Directive Principles act as a guide to the government for making policies and laws for the
purpose of securing justice and welfare.

Source of continuity in Policies:

The Directive Principles are a source of continuity in the policies of the government. In a
democratic system, the governments change after regular intervals and each new
government has to make policies and laws. The presence of Directive Principles ensures that
every government, whether it is formed by a rightist or a leftist party, will exercise its power
for implementing Directive Principles.

Directive Principles are Supplementary to the Fundamental Rights:

Directive Principles are the positive directions to the State for securing and strengthening the
socio-economic dimension of Indian democracy. These aim at the establishment of socio-
economic democracy. These are supplementary to Fundamental Rights which provide for civil
and political rights and freedoms.

Yardstick for measuring the Worth of the Government:

Directive Principles of State Policy constitute a yardstick with which the people can measure
the worth of a government. A government which ignores the task of implementing the
Directive Principles can be rejected by the people in favour of a government by another
political party which can be expected to give due importance to the task of securing the
Directive Principles.

Helpful in the interpretation of the Constitution:

The Directive Principles constitute a manifesto of the aims and goals of the nation. These
reflect the wisdom and views of the founding fathers of the constitution. These reflect the
philosophy of the Constitution and hence provide useful help to the courts in their task of
interpreting the Constitution.

Ambiguity of Directive Principles is Useful:

The Directive Principles have been couched in words which are not very rigid in their
meanings. This ambiguity has been helpful in so far as it helps the State to interpret and apply
these principles in accordance with the socio-economic environment which prevails at a given
time.

Thus, the inclusion of Part IV containing the Directive Principles of State Policy in the
Constitution has been a welcome, worthwhile and useful decision. The Directive Principles
provide for necessary and good foundations for the Indian state as a democratic and welfare
polity. The securing of Directive Principles alone can complete our democratic system,
supplement the Fundamental Rights of the people and build a welfare state characterised by

WWW.IASBABA.COM 46
YWM5YTIyN2MzNDVk

IASBABA’S ILP 2022 – POLITY MODULE 1

Justice, Liberty, Equality and Fraternity. In the words of M.C. Chagla,: “If all these principles
are carried out, our country would indeed be a heaven on earth.”

Q. The ideal of ‘Welfare State’ in the Indian Constitution is enshrined in its (2015)

a) Preamble
b) Directive Principles of State Policy
c) Fundamental Rights
d) Seventh Schedule

Q. Consider the following statements regarding the Directive Principles of State Policy:
(2015)

1. The Principles spell out the socio-economic democracy in the country.


2. The provisions contained in these Principles are not enforceable by any court.
Select the correct answer using the code given below:

a) 1 only
b) 2 only
c) Both 1 and 2
d) Neither 1 nor 2

Q. Consider the following statements:

With reference to the Constitution of India, the Directive Principles of State Policy
constitute limitations upon

1. legislative function.
2. executive function.
Which of the above statements is/are correct?

a) 1 only
b) 2 only
c) Both 1 and 2
d) Neither 1 nor 2

WWW.IASBABA.COM 47
YWM5YTIyN2MzNDVk

IASBABA’S ILP 2022 – POLITY MODULE 1

INTER-RELATIONSHIP BETWEEN FUNDAMENTAL RIGHTS AND DIRECTIVE PRINCIPLES

The question of relationship between the Directive Principles and the Fundamental rights has
caused some difficulty, and the judicial attitude has undergone transformation on this
question over time.

What if a law enacted to enforce a directive principle infringes a fundamental right? On this
question, the judicial view has veered round from irreconcilability to integration between the
Fundamental rights and Directive Principles and in some of the more recent cases, to giving
primacy to the Directive Principles.

Initially, the courts adopted a strict and literal legal position in this respect. The Supreme
Court adopting the literal interpretative approach to Art. 37 ruled that a Directive Principle
could not override a Fundamental right, and that in case of conflict between the two, the
Fundamental right would prevail over the Directive Principle.

The Supreme Court in State of Madras v. Champakam Dorairajan,

“The Directive Principles of the state policy, which by Art. 37 are expressly made
unenforceable by a court cannot override the provisions found in part III (fundamental rights)
which, notwithstanding other provisions, are expressly made enforceable by appropriate
writs, orders or directions under article 32.

The chapter on fundamental rights is sacrosanct and not liable to be abridged by any
legislative or executive act or order, except to the extent provided in the appropriate article
in part III. The Directive Principles of state policy have to conform to and run as subsidiary to
the chapter on Fundamental rights.”

In course of time, The Supreme Court started giving a good deal of value to the Directive
principles from a legal point of view and started arguing for harmonizing the two the
Fundamental rights and Directive Principles.

In Kerala Education Bill the observed “nevertheless, in determining the scope and ambit of
the Fundamental rights relied upon by or on behalf of any person or body, the court may not
entirely ignore these Directive Principles of state policy laid down in part IV of the constitution
but should adopt the principle of harmonious construction and should attempt to give effect
to both as much as possible’’.

The Supreme Court began to assert that there is “no conflict on the whole” between the
fundamental rights and the directive principles. ‘They are complementary and
supplementary to each other’.

In Golak Nath case the Supreme Court there emphasized that the fundamental rights and
directive principles formed an “integrated scheme” which was elastic enough to respond to
the changing needs of the society.

WWW.IASBABA.COM 48
YWM5YTIyN2MzNDVk

IASBABA’S ILP 2022 – POLITY MODULE 1

In Kesavananda Bharti v. State of Kerala, the court observed:

“the fundamental rights and directive principles constitute the “conscience of the
constitution” there is no antithesis between the fundamental rights and directive principles
and one supplements the other.”

“both parts III (fundamental rights) and IV (directive principle) have to be balanced and a
harmonized then alone the dignity of the individual can be achieved they were meant to
supplement each other.,”

The Directive principles and Fundamental rights are not now regarded as exclusionary of each
other. They are regarded as supplementary and complementary to each other. In course of
time, the judicial attitude has veered from irreconcilability to integration of the fundamental
rights and the directive principles. The directive principles which have been declared to be
“fundamental” in the governance of the country cannot be isolated from fundamental rights.
The directive principles have got to be read into the fundamental rights. An example of such
relationship is furnished by the “right to education”.

In Minerva Mills, the court said that the fundamental rights “are not an end in themselves
but are the means to an end.” The end is specified in the directive principles. It was further
observed in the same case that the fundamental rights and directive principles together
“constitute the core of commitment to social revolution and they, together, are the
conscience of the constitution.” The Indian constitution is founded on the bedrock of
“balance” between the two. “To give absolute primacy to one over the other is to disturb the
harmony of the constitution. This harmony and balance between fundamental rights and
directive principles is an essential feature of the basic structure of the constitution.”

In Dalmia Cement, the Supreme Court has emphasized that the core of the commitment of
the constitution to the social revolution through rule of law lies in effectuation of the
fundamental rights and directory principles as supplementary and complimentary to each
other. The preamble to the constitution, fundamental rights and directive principles-the
trinity-are the conscience of the constitution.

The next step in the direction of giving primacy to all directive principles over the fundamental
rights was taken in 1976 when all directive principles were sought to be given precedence
over Arts. 14, 19 and 31 by the 42nd amendment.

But the Supreme Court did not uphold this Amendment as constitutional. The main theme of
the court’s pronouncement was that the constitution is based on the “bedrock of balance”
between the directive principles and fundamental rights and to give absolute primacy to one
over the other would disturb this balance.

Both can co-exist harmoniously. The goals set out in the directive principles are to be achieved
without abrogating the fundamental rights. Both can flourish happily together.

WWW.IASBABA.COM 49
YWM5YTIyN2MzNDVk

IASBABA’S ILP 2022 – POLITY MODULE 1

The principle was restated recently by the Supreme Court in I.R. Coelho v. state of T.N.

“by enacting fundamental rights and directive principles which are negative and positive,
obligations of then states, the constituent assembly made it the responsibility of the
government to adopt a middle path between individual liberty and public good. Fundamental
rights and directive principles have to be balanced. The balanced can be tilted in favour of the
public good. The balance, however, cannot be overturned by completely overriding individual
liberty. This balance is an essential feature of the constitution.

IMPLEMENTATION OF DIRECTIVE PRINCIPLES

Since 1950, the successive governments at the Centre and in the states have made several
laws and formulated various programmes for implementing the Directive Principles. These
are mentioned below:

The successive Five-Year Plans aimed at securing socio-economic justice and reducing
inequalities of income, status and opportunities. In 2015, the Planning Commission was
replaced by a new body called NITI Aayog (National Institution for Transforming India).

Almost all the states have passed land reform laws to bring changes in the agrarian society
and to improve the conditions of the rural masses. These measures include

a) abolition of intermediaries like zamindars, jagirdars, inamdars, etc;


b) tenancy reforms like security of tenure, fair rents, etc;
c) imposition of ceilings on land holdings;
d) distribution of surplus land among the landless labourers; and
e) cooperative farming.

The Minimum Wages Act (1948), the Payment of Wages Act (1936), the Payment of Bonus
Act (1965), the Contract Labour Regulation and Abolition Act (1970), the Child Labour
Prohibition and Regulation Act (1986), the Bonded Labour System Abolition Act (1976), the
Trade Unions Act (1926), the Factories Act (1948), the Mines Act (1952), the Industrial
Disputes Act (1947), the Workmen’s Compensation Act (1923) and so on have been enacted
to protect the interests of the labour sections.

In 2006, the government banned the child labour. In 2016, the Child Labour Prohibition and
Regulation Act (1986) was renamed as the Child and Adolescent Labour Prohibition and
Regulation Act, 1986.

The Maternity Benefit Act (1961) and the Equal Remuneration Act (1976) have been made
to protect the interests of women workers.

Various measures have been taken to utilize the financial resources for promoting the
common good. These include nationalization of life insurance (1956), the nationalization of

WWW.IASBABA.COM 50
YWM5YTIyN2MzNDVk

IASBABA’S ILP 2022 – POLITY MODULE 1

fourteen leading commercial banks (1969), nationalization of general insurance (1971),


abolition of Privy Purses (1971) and so on.

The Legal Services Authorities Act (1987) has established a nation-wide network to provide
free and competent legal aid to the poor and to organize lok adalats for promoting equal
justice. Lok adalat is a statutory forum for conciliatory settlement of legal disputes. It has been
given the status of a civil court. Its awards are enforceable, binding on the parties and final as
no appeal lies before any court against them.

WWW.IASBABA.COM 51
YWM5YTIyN2MzNDVk

IASBABA’S ILP 2022 – POLITY MODULE 1

FUNDAMENTAL DUTIES

INTRODUCTION
• Original Constitution contained only the Fundamental Rights (FRs) and not the
Fundamental Duties (FDs).
• In 1976, the Fundamental Duties were added and in 2002, one more FD was added.
• Inspired by the Constitution of erstwhile USSR
• None of the Constitutions of major democracies contain FDs (except Japan)
• Japanese Constitution is the only democratic country which contains a list of duties of
citizens.
• Socialist countries gave equal importance to the FRs and FDs of the citizens

Mahatma Gandhi in Hind Swaraj observed that “Real rights are a result of the performance
of duty”.

Swaran Singh Committee Recommendations

• In 1976, Congress party set up the Sardar Swaran Singh Committee to make
recommendations about FDs → the need of which was felt during the operation of the
internal emergency (1975 – 1977)
• Committee recommended for the inclusion of FDs in the Constitution.
• It stressed that citizens apart from enjoying the rights also have certain duties to
perform.
• The Congress govt accepted these recommendations and enacted 42nd Constitutional
Amendment Act,1976
• Swaran Singh Committee suggested the incorporation of 8 FDs but 42nd CAA, 1976
included 10 FDs.

Recommendations of Committee which were not included –

1. Parliament may provide for the imposition of penalty/punishment for any non-
compliance with or refusal to observe any duties.
2. No law imposing such penalty/punishment shall be called in question in any court on
the grounds of infringement of any FRs or repugnancy to any other provision of the
Constitution.
3. Duty to pay taxes should also be a FD

42nd Constitutional Amendment Act, 1976

• This amendment added a new part, PART IV A to the Constitution.


• This new part IV A consists of one Article → Article 51 A
• Article 51 A = initially consisted of 10 FDs of the citizens.

WWW.IASBABA.COM 52
YWM5YTIyN2MzNDVk

IASBABA’S ILP 2022 – POLITY MODULE 1

86th Constitutional Amendment Act, 2002

To provide opportunities for education to his child or ward between the age of 6 – 14yrs

LIST OF FDS IN ARTICLE 51 A


1. To abide by the Constitution and respect its ideals and institutions, National Flag and
National Anthem.
2. To cherish and follow the noble ideals that inspired the national struggle for freedom.
3. To uphold and protect the sovereignty, unity and integrity of India.
4. To defend the country and render national service when called upon to do so.
5. To promote harmony and the spirit of brotherhood among all the ppl of India,
linguistic and regional or sectional diversities and to renounce practices derogatory to
the dignity of women.
6. To value and preserve the rich heritage of the county’s composite culture.
7. To protect and improve the natural environment (forests, lakes, rivers and wildlife)
and to have compassion for living creatures.
8. To develop scientific temper, humanism and the spirit of inquiry and reform.
9. To safeguard public property and to adjure violence.
10. To strive towards excellence in all spheres of individual and collective activity so that
the nation constantly rises to higher level of endeavor and achievement.
11. To provide opportunities for education to his child or ward between the age of 6 –
14yrs → added in 86th Constitutional Amendment Act, 2002.

Amendment of article 51A.- In article 51A of the Constitution, after clause (J), the following
clause shall be added, namely: -

"(k) who is a parent or guardian to provide opportunities for education to his child or, as the
case may be, ward between the age of six and fourteen years.".

FEATURES OF THE FUNDAMENTAL DUTIES


1. Some of them are Moral Duties (Ex. Cherishing noble ideals of freedom struggle)
2. Some of them are Civic Duties (Ex. Respecting the institution, National Flag/Anthem)
3. They refer to such values which have been a part of the Indian tradition, mythology,
religions and practices.
4. Essentially contain just a codification of tasks integral to the Indian way of life.
5. FDs are confined to citizens only, do not extend to foreigners (FRs = extend to all
→Citizens + Foreigners)
6. Like DPSPs, FDs are also non-justiciable → Constitution doesn’t provide for the direct
enforcement by the courts.
7. There is no legal sanction against their violation (however the Parliament is free to
enforce them by suitable legislation)

WWW.IASBABA.COM 53
YWM5YTIyN2MzNDVk

IASBABA’S ILP 2022 – POLITY MODULE 1

Q. Which of the following is/are among the Fundamental Duties of citizens laid down in
the Indian Constitution? (2015)

1. To preserve the rich heritage of our composite culture.


2. To protect the weaker sections from social injustice.
3. To develop the scientific temper and spirit of inquiry.
4. To strive towards excellence in all spheres of individual and collective activity.
Select the correct answer using the codes given below:

a) 1 and 2 only
b) 2 only
c) 1, 3 and 4 only
d) 1, 2, 3 and 4

Q. Under the Constitution of India, which one of the following is not a Fundamental Duty?
(2011)

a) To vote in public elections.


b) To develop the scientific temper.
c) To safeguard public property.
d) To abide by the Constitution and respect its ideals

Q. Which of the following statements is/are true of the Fundamental Duties of an Indian
citizen?

1. A legislative process has been provided to enforce these duties.


2. They are correlative to legal duties.
Select the correct answer using the code given below:

a) 1 only
b) 2 only
c) Both 1 and 2
d) Neither 1 nor 2

Q. In the Constitution of India, promotion of international peace and security is included


in the (2014)

a) Preamble to the Constitution


b) Directive Principles of State Policy
c) Fundamental Duties
d) Ninth Schedule

WWW.IASBABA.COM 54
YWM5YTIyN2MzNDVk

IASBABA’S ILP 2022 – POLITY MODULE 1

CRITICISM OF FUNDAMENTAL DUTIES


1. List of duties is not exhaustive; it doesn’t cover important duties like casting vote,
paying taxes, family planning and so on.
2. Some of the duties are vague, ambiguous and difficult to understand (Ex. Phrases like
Noble ideals, scientific temper etc.)
3. They have been described by the critics as – code of moral precepts due to their non-
justiciable character.
4. Their inclusion in the constitution is superfluous: this is because the duties included in
the Constitution would be performed by the people even though they were not
incorporated in the Constitution.
5. Inclusion of FDs as an appendage to Part IV has reduced their value and significance;
they should have been added after part III so as to keep them on par with FRs.

SIGNIFICANCE OF FUNDAMENTAL DUTIES


• It serves as a reminder to the citizens that while enjoying their rights, they should also
be conscious of duties they owe to their country, their society and fellow citizens.
• It serves as a warning against the national and antisocial activities (destroying pub
property/burning national flag)
• Serve as a source of inspiration and promotes discipline and commitment; creates a
feeling that citizens are not mere spectators but active participants in the realization
of national goals.
• They help the courts in determining the constitutional validity of a law.
• They are enforceable by law. Hence, Parliament can provide for the imposition of
penalty/punishment for failure to fulfill any of them.
• Their inclusion helps in strengthening the democracy.

Verma Committee Observations (1999) identified the existence of legal provisions in line with
FDs

• The Prevention of Insults to National Honour Act (1971) → prevents disrespect to the
Constitution, National Flag and National Anthem.
• Various Criminal Laws →punishes for encouraging enmity between different sections
of people.
• The Protection of Civil Rights, 1955 or The Untouchability (Offences) Act, 1976 →
punishes for offences related to caste and religion.
• Indian Penal Code (IPC) → declares the imputations and assertions prejudicial to
national integration as punishable offences.
• The Unlawful Activities (Prevention) Act, 1967 → provides for the declaration of a
communal organization as an unlawful association.

WWW.IASBABA.COM 55
YWM5YTIyN2MzNDVk

IASBABA’S ILP 2022 – POLITY MODULE 1

• The Representation of Peoples Act, 1951 → provides for disqualification of MPs/MLAs


for indulging in corrupt practices or soliciting votes on the grounds of religion or
promoting enmity between different sections of ppl.
• The Wildlife (Protection) Act, 1972 → prohibits trade in rare and endangered species.
• The Forest (Conservation) Act, 1980 → checks indiscriminate deforestation and
diversion of forest lands for non-forest purposes.

FUNDAMENTAL RIGHTS AND FUNDAMENTAL DUTIES


As provided in Article 32 of the Constitution (which itself is a fundamental right) fundamental
rights are enforceable through Supreme Court. High Court also under Article 226 can issue
Writs etc. for enforcement of fundamental rights. However, Fundamental Duties are not
enforceable through courts.

The Courts while interpreting Fundamental Rights or any restrictions imposed on such rights
may take into account the Fundamental Duties and also the Directive Principles of the State
policy enshrined in Part IV of the Constitution.

In State of Gujarat v. Mirzapur (2005) while considering provisions of Articles 48, 48-A and
also Article 51-A(g), the Supreme Court held:

“. It is thus clear that faced with the question of testing the constitutional validity of any
statutory provision or an executive act, or for testing the reasonableness of any restriction cast
by law on the exercise of any fundamental right by way of regulation, control or prohibition,
the directive principles of State policy and fundamental duties as enshrined in Article 51-A of
the Constitution play a significant role.”

In Ramlila Maidan Incident, (2012) it was held:

“There has to be a balance and proportionality between the right and restriction on the one
hand, and the right and duty, on the other. It will create an imbalance, if undue or
disproportionate emphasis is placed upon the right of a citizen without considering the
significance of the duty. The true source of right is duty. When the courts are called upon to
examine the reasonableness of a legislative restriction on exercise of a freedom, the
fundamental duties enunciated under Article 51-A are of relevant consideration. Article 51-A
requires an individual to abide by the law, to safeguard public property and to abjure violence.
It also requires the individual to uphold and protect the sovereignty, unity and integrity of the
country. All these duties are not insignificant.’’

Fundamental Rights guaranteed under Part III of the Constitution are important natural rights
necessary for development of human beings. They are enforceable through court of law. No
law can be made which takes away or abridges any fundamental rights. On the other hand,
Fundamental Duties though not enforceable, but always taken into account while
interpreting any fundamental rights.

WWW.IASBABA.COM 56
YWM5YTIyN2MzNDVk

IASBABA’S ILP 2022 – POLITY MODULE 1

Q. In the context of India, which one of the following is the correct relationship between
Rights and Duties?

a) Rights are correlative with Duties.


b) Rights are personal and hence independent of society and Duties.
c) Rights, not Duties, are important for the advancement of the personality of the
citizen.
d) Duties, not Rights, are important for the stability of the State.

WWW.IASBABA.COM 57
YWM5YTIyN2MzNDVk

IASBABA’S ILP 2022 – POLITY MODULE 1

AMENDMENT OF THE CONSTITUTION

INTRODUCTION
Constitution provides for its amendment → in order to adjust itself to the changing conditions
and needs.

Indian Constitution is neither flexible nor rigid but a synthesis of both.

Article 368 in Part XX →deals with the powers of the Parliament to amend the Constitution
and its procedure.

Article 368 states that “the Parliament may amend by way of addition, variation or repeal any
provision of the Constitution in accordance with the procedure laid down for the purpose”.

However, the Parliament cannot amend those provisions which form the ‘basic structure’ of
the Constitution → ruled by the Supreme Court in the Kesavananda Bharathi vs. State of
Kerala (1973) case.

PROCEDURE FOR AMENDMENT AS LAID DOWN IN ARTICLE 368 PART XX


1. Amendment of the Constitution can be initiated only by the introduction of a bill in
either of the House and not in the state legislatures.
2. The Bill can be introduced either by a minister or by a private member and doesn’t
require prior permission of the President.
3. The bill must be passed in each House by a Special Majority**
4. Each House must pass the bill separately. In case of a disagreement between the 2
Houses, there is no provision for holding a joint sitting of the 2 Houses.
5. If the bill seeks to amend the federal provisions of the Constitution, it must also be
ratified by the state legislatures of half of the states by a simple majority#
6. After duly passed by both Houses and ratified by the state legislatures, the Bill is
presented to the President for assent.
7. The President must give his assent; he can neither withhold his assent to the bill nor
return the bill for reconsideration of the Parliament.
8. After the President’s assent the Bill becomes an Act (CAA)

TYPES OF AMENDMENTS

SIMPLE MAJORITY
Simple majority or working majority refers to majority of more than 50% of the members
present and voting. Example:

• Total strength of Lok Sabha: 545


• Vacant Seats: 5
• Members present: 500
• Members present, but decide to abstain / not to vote: 50
WWW.IASBABA.COM 58
YWM5YTIyN2MzNDVk

IASBABA’S ILP 2022 – POLITY MODULE 1

• Members present and voting: 500-50=450


• Simple Majority in this case would be: 226
• Most of the normal motions and bills in the house such as No-confidence Motion,
Motion of Confidence, Motion of Thanks, Censure Motion, Adjournment Motion,
Money Bills, Ordinary Bills etc.

ABSOLUTE MAJORITY
Absolute majority refers to the majority of more than 50% of the total strength of the house.
Example:

• Total strength of Lok Sabha: 545


• Absolute Majority: 273
• Such kind of majority is not required in isolation in the Indian Parliament. There are
instances when such majority is needed with other majority which would be thus
called special majority.

EFFECTIVE MAJORITY
Effective Majority of house means more than 50% of the effective strength of the house. This
implies that out of the total strength, we deduct the absent and vacant seats.

• Total strength of Lok Sabha: 545


• Vacant Seats: 5
• Effective Strength: 545-5=540
• Members present, but decide to abstain / not to vote: 50
• Members present and voting: 540-50=490
• Effective Majority: 490/2+1=245
• In constitution of India, the “all the then members” present indicates an effective
majority. In Constitution, effective majorities are needed for removal of Vice-
President, Deputy Chairman of Rajya Sabha, Lok Sabha speaker and Deputy Speaker.

SPECIAL MAJORITY
Any Majority other than simple, absolute and effective majority is called special majority.
These include

• Majority by two-third strength of the house {example impeachment of president


under article 61}
• Majority by two-third of present and voting members {Example: Power of Parliament
to legislate with respect to a matter in the State List in the national interest, under
article 249}; certain constitution amendment bills etc.
• Absolute majority + majority of two-third present and voting {Example: Removal of
Supreme Court Judge, CAG etc.}

WWW.IASBABA.COM 59
YWM5YTIyN2MzNDVk

IASBABA’S ILP 2022 – POLITY MODULE 1

EXAMPLES OF MAJORITIES IN CONSTITUTION

Impeachment of President: Special Majority

According to Article 61, When a President is to be impeached for violation of the Constitution;
the charge shall be preferred by either House of Parliament. A 14 days’ notice to move a
resolution is given. Then, the resolution has to be passed by a majority of not less than two-
thirds of the total membership of the House. This is an example of Special Majority.

Removal of the Vice-President: Effective Majority

Vice-President may be removed from his office by a resolution of Rajya Sabha passed by a
majority of all the then members of the Rajya Sabha and agreed to Lok Sabha. This is an
example of effective majority in Rajya Sabha.

Removal of Deputy chairman Rajya Sabha: Effective Majority

A member holding office as Deputy Chairman of Rajya Sabha may be removed from his office
by a resolution of the Council passed by a majority of all the then members of the Council.
(Simple Majority in Rajya Sabha)

Removal of Speaker and Lok Sabha Speaker: Effective Majority

Member holding office as Speaker or Deputy Speaker of the House of the People may be
removed from his office by a resolution of the House of the People passed by a majority of all
the then members of the House.

Removal of Supreme Court Judge: Absolute + Special Majority

A Judge of the Supreme Court shall not be removed from his office except by an order of the
President passed after an address by each House of Parliament supported by a majority of
the total membership of that House (Absolute Majority) and by a majority of not less than
two-thirds of the members of that House present and voting (Special Majority) voting has
been presented to the President in the same session for such removal on the ground of
proved misbehavior or incapacity. (Article 124)

Abolition of Council of States: Absolute + Special Majority

Parliament may by law provide for the abolition of the Legislative Council of a State having
such a Council or for the creation of such a Council in a State having no such Council, if the
Legislative Assembly of the State passes a resolution to that effect by a majority of the total
membership of the Assembly (Absolute Majority) and by a majority of not less than two-thirds
of the members of the Assembly present and voting. (Special Majority) Article 169. (1)

WWW.IASBABA.COM 60
YWM5YTIyN2MzNDVk

IASBABA’S ILP 2022 – POLITY MODULE 1

Removal of Speaker or Deputy Speaker of Assembly: (Effective Majority)

Speaker or Deputy Speaker of Assembly may be removed from his office by a resolution of
the Assembly passed by a majority of all the then members of the Assembly (Effective
Majority). Article 179 (C)

Removal of Chairman or Deputy Chairman of a Legislative Council: (Effective Majority)

Chairman or Deputy Chairman of a Legislative Council may be removed from his office by a
resolution of the Council passed by a majority of all the then members of the Council. (Simple
Majority) Article 183 (C)

Emergency Proclamation (Absolute + Special Majority)

According to article 352 (4) an emergency proclamation is laid before each House of
Parliament and shall cease to operate at the expiration of one month unless before the
expiration of that period it has been approved by resolutions of both Houses of Parliament.
Once approved it shall cease to be in force if again not approved within six months. For both
of these purposes, the resolution should be passed by either House of Parliament only by a
majority of the total membership of that House (Absolute Majority) and by a majority of not
less than two-thirds of the Members of that House present and voting. (Special Majority)

Amendment of the Constitution via article 368: (Absolute + Special Majority)

According to Article 368(2), amendment to Constitution may be initiated only by the


introduction of a Bill for the purpose in either House of Parliament, and when the Bill is passed
in each House by a majority of the total membership of that House (Absolute Majority) and
by a majority of not less than two-thirds of the members of that House present and voting,
(special Majority).

Further, if the amendment of the constitution also requires the assent of the state assemblies,
they can pass the constitutional Amendment Bill with simple majority.

1. By Simple Majority of the Parliament – outside the scope of Article 368

1. Admission or establishment of new states.


2. Formation of new states or alteration of areas, boundaries or names of existing states.
3. Abolition or creation of legislative council in states.
4. 2nd Schedule → emoluments, allowances, privileges etc. of the
President/Governor/Speakers/Judges.
5. Quorum in Parliament.
6. Salaries and allowances of MPs.
7. Rules of procedure in Parliament.
8. Privileges of the Parliament, its members and its committees.
9. Use of English language in Parliament.
10. Number of puisne judges in SC.

WWW.IASBABA.COM 61
YWM5YTIyN2MzNDVk

IASBABA’S ILP 2022 – POLITY MODULE 1

11. Conferment of more jurisdictions on the SC.


12. Use of official language.
13. Citizenship -- acquisition and termination.
14. Election to the Parliament and State legislatures.
15. Delimitation of constituencies.
16. Union Territories.
17. 5th Schedule → administration of scheduled areas and scheduled tribes.
18. 6th Schedule → administration of tribal areas.

2. By Special Majority of the Parliament

1. FRs
2. DPSPs
3. All other provision which are not covered under (1) and (3)

3. By Special Majority of the Parliament and the ratification of half of state legislatures

1. Election of the President and its manner.


2. Extent of the executive power of the Union and the states.
3. SCs and HCs.
4. Distribution of legislative powers between the Union and the states.
5. Any of the lists in 7th Schedule.
6. Representation of states in Parliament.
7. Power of Parliament to amend the Constitution and its procedure (Article 368 itself)

AMENDABILITY OF FRS

The question whether FRs can be amended by the Parliament under Article 368 came for
consideration of the SC →

Shankari Prasad v. Union of India (1951) Case – the constitutional validity of the 1st
Amendment Act (1951) which curtailed the Right to Property was challenged.

SC ruled that – “the power of the Parliament to amend the Constitution under Article 368 also
includes the power to amend FRs”

Golak Nath v. State of Punjab (1967) Case – the constitutional validity of the 7th Amendment
Act which inserted certain state acts in 9th Schedule was challenged.

SC ruled that –“FRs are given a transcendental and immutable position and hence the
Parliament cannot abridge or take away any of the FRs”.

The Parliament in response to Golak Nath Case, enacted the 24th CAA (1971) → declared that
“the Parliament can abridge or take away any of the FRs under Article 368”.

WWW.IASBABA.COM 62
YWM5YTIyN2MzNDVk

IASBABA’S ILP 2022 – POLITY MODULE 1

Kesavananda Bharathi v. State of Kerala (1973) case – SC overruled its judgment in the Golak
Nath Case (1971) and laid down a new doctrine of ‘basic structure’.

SC upheld the validity of 24th CAA (1971) and stated that -- “the Parliament is empowered
to abridge or take away any of the FRs under Article 368 but does not enable the Parliament
to alter the ‘basic structure’ of the Constitution”.

[This means → Parliament cannot abridge or take away a FR that forms a part of the ‘basic
structure’ of the Constitution]

The Parliament in response to this judicially innovated doctrine of ‘basic structure’, enacted
the 42nd CAA (1976) → amended Article 368 and declared that “there is no limitation on the
constituent power of Parliament and no amendment can be questioned in any court on any
ground including the contravention of any FRs”

Minerva Mills v. Union of India (1980) – invalidated this provision as it excludes Judicial
Review which is a ‘basic feature’ of the Constitution.

Waman Rao v. Union of India (1981) – SC adhered to the doctrine of ‘basic structure’ and
further clarified that it would apply to constitutional amendments enacted after April 24,
1973 (i.e., the Date of judgment in the Kesavananda Bharathi case).

The present position is that the Parliament under Article 368 can amend any part of the
Constitution including the FRs but without affecting the ‘basic structure’ of the Constitution.

The following have emerged as ‘basic structures’ from various judgments –

1. Supremacy of the Constitution.


2. Sovereign, democratic and republican nature of the Indian Polity.
3. Secular character of the Constitution.
4. Separation of powers between the L, E and J.
5. Federal character of the Constitution.
6. Unity and integrity of the Nation.
7. Welfare State (socio – economic justice)
8. Judicial Review
9. Harmony and balance between FRs and DPs
10. Parliamentary System
11. Rule of Law
12. Freedom and dignity of the individual
13. Free and fair elections
14. Principle of Equality
15. Independence of Judiciary
16. Effective access to justice
17. Limited power of Parliament to amend Constitution.
18. Reasonableness.

WWW.IASBABA.COM 63
YWM5YTIyN2MzNDVk

IASBABA’S ILP 2022 – POLITY MODULE 1

CRITICISM OF THE AMENDMENT PROCEDURE


1. No provision of special body like Constitutional Convention (as in USA) or Constituent
Assembly for amending the Constitution.
2. The constituent power is vested in the Parliament and only in few cases in the state
legislatures (that too very minimal role).
3. The power to initiate an amendment lies with Parliament (unlike in USA); the state
legislatures cannot initiate amendment bill except in 1 case.
4. SL can pass a resolution requesting the Parliament to create or abolish legislature
councils in the states → here also; the Parliament can either approve or disapprove
such a resolution.
5. Major part of the Constitution can be amended by the Parliament itself and only in
few cases it needs ratification of states.
6. Constitution does not prescribe the time frame within which the state legislatures
should ratify or reject an amendment submitted to them.
7. Also, it is silent on the issue whether the states can withdraw their approval after
according the same.
8. No provision of holding Joint Sitting of the Houses.
9. The process of amendment is similar to that of a legislative process. Except for the
special majority, the amendment bills are to be passed by the Parliament in the same
way as ordinary bills.
10. The provisions relating to the amendment procedures are too sketchy. They leave a
wide scope for taking the matters to the judiciary.

**Special Majority – a majority (that is, more than 50%) of the total membership of each
House and a majority of two – thirds of the members of each House present and voting.

The expression ‘Total Membership’ means the total no. of members comprising the House
irrespective of the fact whether there are absentees or vacancies.

#Simple Majority – a majority of the House present and voting.

IMPORTANT AMENDMENTS

Amendment Act Newly added, Removed and Amendments


Amended
Articles/Scheduled/Parts

7th Amendment Article 1 The Seventh Amendment brought


Act 1956 about the most comprehensive
Article 3
changes so far in the Constitution. This
amendment was designed to

WWW.IASBABA.COM 64
YWM5YTIyN2MzNDVk

IASBABA’S ILP 2022 – POLITY MODULE 1

Article 49 implement the State Reorganisation


Act.
Article 80
• Reorganisation of states on
Article 81
linguistic basis
Article 82 • Abolition of Class A, B, C and D
Article 131 states
• Introduction of Union Territories
Article 153
The Second and Seventh schedules
Article 158 were substantially amended for the
Article 168 purpose of the States Reorganization
Act.
Article 170

Article 171

Article 216

Article 217

Article 220

Article 222

Article 224

Article 230

Article 231

Article 232

Part VIII

First, Second, Fourth and


Seventh Schedules of Indian
Constitution

9th Amendment Schedule 1 of Indian It provided for the transfer of certain


Act, 1960 Constitution territories of India to Pakistan under an
agreement between India and Pakistan
as a part of a comprehensive
settlement of border disputes between
the two countries.

WWW.IASBABA.COM 65
YWM5YTIyN2MzNDVk

IASBABA’S ILP 2022 – POLITY MODULE 1

10th Article 240 Integrated the areas of Free Dadra and


Amendment Act, Nagar Haveli with the Union of India
First Schedule
1961 and provides for their administration
under the regulation making powers of
the President

12th Article 240 The main object of the Amendment


Amendment Act was to add Union Territories of Goa,
First Schedule
1961 Daman and Diu to the Union of India
and for this First Schedule of the
Constitution was amended.

13th Article 170 The Act provides the creation of


Amendment Act, Nagaland as the Sixteenth State of the
Added new article 371A
1963 Union.

The Amendment provides also for the


vesting of certain special
responsibilities in the Governor of
Nagaland. (under Article 371A)

14th Articles 81 and 240 The amendment provides for the


Amendment Act, incorporation of the former French
First and fourth Schedules
1962 Establishments in India, under the
Added Article 239A name Pondicherry, as an integral part
of the territory of the Indian Union.

It also amended Article 31 to increase,


from a maximum 20 to 25, the number
of seats assigned in the Lok Sabha for
the Union Territories.

17th Amendment Amend article 31A. To secure the constitutional validity of


Act, 1964 acquisition of Estates and place land
Amend schedule 9
acquisition laws in Schedule 9 of the
constitution.

21st Amendment Eighth Schedule It amended the Eighth Schedule to the


Act, 1967 Constitution by including ‘Sindhi’
therein.

24th Amended Article 13 and 368 It amended Article 13 and 368 with a
Amendment Act view to removing all possible doubts
1971 regarding the power of Parliament to

WWW.IASBABA.COM 66
YWM5YTIyN2MzNDVk

IASBABA’S ILP 2022 – POLITY MODULE 1

amend the Constitution and procedure


thereof.

It gets over the Golak Nath ruling and


asserts the power of Parliament,
denied to in the Golak Nath, to amend
Fundamental Rights.

25th Added a new clause, Article Upto 1971, the position was that
Amendment Act 31C Fundamental Rights prevailed over the
1971 Directive Principles of State Policy and
that a law enacted to implement a
Directive Principle could not be valid if
it conflicted with a Fundamental Right.

Article 31C sought to change this


relationship to some extent by
conferring primacy on Articles 39(b)
and 39(c) over Articles 14, 19 and 31.

26th Article 366 The cumulative effect of these changes


Amendment Act was the end of the recognition granted
Added Article 363A
1971 to the former rulers of Indian States
Removed Articles 291 and and the abolition of Privy Purses.
362

36th Articles 80 and 81 Sikkim incorporated as an Indian state.


Amendment Act
First and fourth Schedules
40th
Added Article 371F
Amendment Act
1975 Removed Article 2A

42nd Article 31 The Act inter-alia gave preponderance


Amendment Act to the Directive Principles of State
Article 31C
1976 Policy over the Fundamental Rights.
Article 39
Established the supremacy of
Article 55 Parliament and curtailed the powers of
Judiciary. The Act was first of its kind.
Article 74
It was the most comprehensive Act and
Article 77
touched almost all the sensitive areas
Article 81 of the Constitution.

WWW.IASBABA.COM 67
YWM5YTIyN2MzNDVk

IASBABA’S ILP 2022 – POLITY MODULE 1

Article 82 The Amendment was meant to


enhance enormously the strength of
the Government.
Article 83
The major Amendments made in the
Article 100 Constitution by the 42nd Amendment
Act are:
Article 102
Preamble
Article 103
The characterization of India as
Article 105
‘Sovereign Democratic Republic’ has
Article 118 been changed to ‘Sovereign Socialist
Secular Democratic Republic’. The
Article 145
words ‘Unity of the nation’ have been
Article 150 changed to ‘Unity and integrity of the
Article 166 nation’.

Article 170 Parliament and State Legislatures :


The life of the Lok Sabha and State
Article 172 Legislative Assemblies was extended
Article 189 from 5 to 6 years.

Article 191 Executive : It amended Article 74 to


State explicitly that the President shall
Article 192 act in accordance with the advice of the
Article 194 Council of Ministers in discharge of his
functions.
Article 208
Judiciary: The 42nd Amendment Act
Article 217 inserted Article 32A in order to deny
Article 225 the Supreme Court the power to
consider the Constitutional validity of a
Article 226 State law. Another new provision,
Article 227 Article 131A, gave the Supreme Court
an exclusive jurisdiction to determine
Article 228
question relating to the Constitutional
Article 311 validity of a central laws.

Article 312 Article 144A and Article 128A, the


creatures of Constitutional
Amendment Act made further
Article 330 innovation in the area of judicial review
of the Constitutionality of legislation.

WWW.IASBABA.COM 68
YWM5YTIyN2MzNDVk

IASBABA’S ILP 2022 – POLITY MODULE 1

Article 352 Under Article 144A, the minimum


number of judges of the Supreme Court
Article 353
to decide a question of a Constitutional
Article 356 validity of a Central or State law was
fixed as at least seven and further, this
Article 357
required two-thirds majority of the
Article 358 judges sitting declare law as
unconstitutional. While the power of
Article 359
the High Court to enforce Fundamental
Article 366 Rights remained untouched, several
Article 368 restrictions were imposed on its power
to issue writs ‘for any other purpose’.
Article 371F
Federalism: The Act added Article 257A
Seventh Schedule in the Constitution to enable the
Added Articles 31D, 32A, 39A, Centre to deploy any armed force of
43A, 48A, 131A, 139A, 144A, the Union, or any other force under its
226A, 228A and 257A control. For dealing with any grave
situation of law and order in any State.
Added Parts IVA and XIVA
Fundamental Rights and Directive
Principles: A major change that was
made by 42nd Constitutional
Amendment was to give primacy to all
Directive Principles over the
Fundamental Rights contained in
Articles 14, 19 or 31. The 42nd
Constitutional Amendment added a
few more Directive Principles - free
legal aid, participation of workers in
management of industries, protection
for environment and protection of
forests and wildlife of the country.

Fundamental Duties: The 42nd


Amendment Act inserted Article 51-A
to create a new part called IV-A in the
Constitution, which prescribed the
Fundamental Duties to the citizens.

Emergency : Prior to 42nd Amendment


Act, the President could declare
emergency under Article 352

WWW.IASBABA.COM 69
YWM5YTIyN2MzNDVk

IASBABA’S ILP 2022 – POLITY MODULE 1

throughout the country and not in a


part of the country alone. The Act
authorized the President to proclaim
Emergency in any part of the country.

The dominant thrust of the


Amendment was to reduce the role of
courts, particularly, that of the High
Courts. It also sought to strengthen
Parliament in various ways which in
effect, added to the power of the
Central Government. It drew enormous
criticism particularly for it was pushed
through during Emergency.

44th Article19 The 44th Amendment passed in 1978


Amendment Act undid most of the distortions
Article 22
1978 introduced into the Constitution by the
Article 30 42nd Amendment of the Constitution.

Article 31A The salient features of the


Amendment Act are as follows :
Article 31C
• It reduced the life of Lok Sabha and
Article 38
State Legislative Assemblies again
Article 71 to five years and thus restored the
Article 74 status quo ante.

Article 77 • It cancelled 39th Amendment


which had deprived the Supreme
Article 83 Court of its jurisdiction to decide
Article 103 disputes concerning election of the
President and the Vice-President.
Article 105
• A new provision was added to
Article 123
Article 74(1) saying that the
Article 132 President could require the council
of ministers to reconsider its advice
Article 133
to him, either generally or
Article 134 otherwise and the President should
Article 139A Act in accordance with the advice
tendered after such re-
Article 150 consideration.

WWW.IASBABA.COM 70
YWM5YTIyN2MzNDVk

IASBABA’S ILP 2022 – POLITY MODULE 1

Article 166 Article 257A was Omitted

Article 172 • It has been provided that an


Emergency can be proclaimed only
Article 192
on the basis of written advice
Article 194 tendered to the President by the
Cabinet.
Article 213

Article 217 • Right to Property has been taken


out from the list of Fundamental
Article 225 Rights and has been declared a
Article 226 legal right.

Article 227

Article 239B

Article 329

Article 352

Article 356

Article 358

Article 359

Article 360

Article 371F

Ninth Schedule

Added Articles 134A and


361A

Deletion of Articles 31, 257A


and 329A

53rd The amendment Act elevated the


Amendment Act Union Territory of Mizoram to the
1986 status of a State.

55th Amendment The Union Territory of Arunchal


Act 1986 Pradesh was elevated to the status of a
State by the 55th Amendment Act.

WWW.IASBABA.COM 71
YWM5YTIyN2MzNDVk

IASBABA’S ILP 2022 – POLITY MODULE 1

61st Amendment Article 326 The 61st Amendment reduces the


Act 1989 voting age from 21 years to 18 years for
the Lok Sabha and Assembly election.

69th Insertion of new articles 239 The Amendment Act was to grant
Amendment Act AA and 239 AB Statehood to Delhi as ‘National Capital
1991 Territory of Delhi’.

Confers special previsions for Delhi


(National Capital Territory of Delhi)

It also provides a 70 member assembly


and a 7 member Council of Ministers
for Delhi

71st Amendment Article 332 Manipuri, Konkani, and Nepali were


Act 1992 added in the 8th Schedule of Indian
Constitution

73rd Added Part IX Introduction of Panchayat Raj


Amendment Act
Added 11th Schedule Addition of Part IX and Eleventh
1992
Schedule to the Indian Constitution
Article 243A
The Act provides for Gram Sabha, a
three-tier model of Panchayati Raj,
reservation of seats for SCs and STs in
proportion to their population and
one-third reservation of seats for
women.

74th Article 280 Article 280 Introduction of Municipalities and


Amendment Act Nagarpalikas
Added Part IXA
1992
The Act provides Municipal Panchayat,
Added 12th Schedule
Municipal Council and Municipal
Corporation, reservation of seats for
SCs and STs in proportion to their
population and one-third reservation
of seats for women

77th added a new clause (4-a) to Empowers the State to make any
Amendment Act Article 16 provisions for reservation in
1995 promotions in Government jobs in

WWW.IASBABA.COM 72
YWM5YTIyN2MzNDVk

IASBABA’S ILP 2022 – POLITY MODULE 1

favour of SCs and STs, if it is of opinion


that they are inadequately represented
in the services under the State.

This has been done to nullify the effect


of the Supreme Court Judgment in the
Mandal Commission Case (Indra
Sawhney vs. Union of India) in which
the Court has held that reservation in
promotions cannot be made.

86th Amended Articles 45 and 51A With a view to making right to free and
Amendment Act compulsory education a fundamental
Added Article 21A
2002 right, the Act inserts a new Article,
The Act amends in Part-III, namely, Article 21A conferring on all
Part –IV and Part-IV (A) of the children in the age group of 6 to 14
Constitution. years the right to free and compulsory
education.

89th Added Article 338A Creation of National Commission for


Amendment Act Scheduled Tribes.
2003

92nd 8th Schedule The Amendment facilitates for the


Amendment Act inclusion of Bodo, Dogari, Maithili and
2003 Santhali in the VIIIth Schedule of the
Constitution. With the inclusion of
these four languages, the number of
languages in the VIIIth Schedule goes
upto 22.

93rd The 93rd Amendment enables


Amendment Act provision of reservation (27%) for OBCs
2006 in government as well as private
educational institutions.

95th Article 334 Extension of reservation of seats for


Amendment Act SC/ST
2010
Nomination of Anglo-Indian members
in Parliament and State Assemblies to
Seventy years

WWW.IASBABA.COM 73
YWM5YTIyN2MzNDVk

IASBABA’S ILP 2022 – POLITY MODULE 1

96th Eighth Schedule It has substituted "Odia" for "Oriya" in


Amendment Act the 8th Schedule
2011

97th Articles 19 Introduction of Part IXB in the


Amendment Act Constitution of India relating to Co-
Added Part IXB
2012 operative Societies

99th Insertion of new articles The amendment provides for the


Amendment Act 124A, 124B and 124C. formation of a National Judicial
2015 Appointments Commission.
Amendments to Articles 127,
128, 217, 222, 224A, 231. The amendment was struck down by
the Supreme Court on 16 October
2015.

100th Amendment of First Schedule Exchange of some enclave territories


Amendment Act with Bangladesh
2015
Conferment of citizenship rights to
citizens of enclaves resulting to signing
of Land Boundary Agreement (LBA)
Treaty between India and Bangladesh.

101st Article 248 Introduction of Goods and Services Tax


Amendment Act (GST)
Article 249
2016
Article 250

Article 268

Article 269

Article 270

Article 271

Article 286

Article 366

Article 368

Amended Sixth Schedule and


Seventh Schedule

Deletion of Article 268A

WWW.IASBABA.COM 74
YWM5YTIyN2MzNDVk

IASBABA’S ILP 2022 – POLITY MODULE 1

102nd Addition of articles 338B, Constitutional status to National


Amendment Act 342A, and Added Clause 26C. Commission for Backward Classes
2018
Modification of articles 338,
366

103rd Amendment to Article 15, A maximum of 10% Reservation for


Amendment Act added Clause [6] Economically Weaker Sections (EWSs).
2019
Amendment to Article 16,
added Clause [6]

104th Amend article 334 To extend the reservation of seats for


Amendment Act SCs and STs in the Lok Sabha and states
2020 assemblies from Seventy years to
Eighty years. Removed the reserved
seats for the Anglo-Indian community
in the Lok Sabha and state assemblies.

Q. Consider the following statements: (2013)

1. An amendment to the Constitution of India can be initiated by an introduction of a


bill in the Lok Sabha only.
2. If such an amendment seeks to make changes in the federal character of the
Constitution, the amendment also requires to be ratified by the legislature of all the
States of India.
Which of the statements given above is/are correct?

a) 1 only
b) 2 only
c) Both 1 and 2
d) Neither 1 nor 2

WWW.IASBABA.COM 75
YWM5YTIyN2MzNDVk

IASBABA’S ILP 2022 – POLITY MODULE 1

BASIC STRUCTURE OF THE CONSTITUTION

INTRODUCTION
Constitution of India did not emerge from vacuum. It is continuous process of evolution,
reformation and recreating the existing system of governance by eminent scholars, experts
and judges etc. No Constitution can remain static. It must respond to new challenges and take
account of unanticipated and unforeseen events which were not within the contemplation of
the framers of the Constitution.

Ours is the living Constitution which requires an amendment from time to time according to
the societal changes. Parliament in its constituent power can amend by way of addition,
alteration, variation or repeal any provisions of the Constitution. On its plain terms Art.368
is plenary and is not subject to any limitations or exceptions. The Constituent Assembly
debates indicate that the founding fathers did not envisage any limitation on the amending
power.

Bringing alteration to the Constitution provisions by the Parliament was very easy process
before Kesavananda Bharathi’s Case, because there was no implied or express limitation on
its amending power exercised under the Constitution. But in the Kesavananda case,
uncontrolled power of the Parliament has been controlled and curtailed by the Doctrine of
Basic Structure.

We did not have this doctrine at the commencement of the Constitution of India. This
doctrine conceived in the case of Sajjan Singh and took real birth in the case of Kesavananda
Bharathi’s Case. It is the product of long struggle between the Judiciary and Parliament.
Through this basic structure principle, the Supreme Court changed the course of
Constitutional history by denying the assertion of supremacy of Parliament in matter of
amending the Constitution at solely on the basis of requisite voting strength, quite unmindful
of the basic or fundamental rights of citizens.

Art.31-B and Ninth Schedule are the main root cause for developing this doctrine by the
Judiciary in so many cases. The reason is, this Schedule made controlled Constitution into
uncontrolled by excluding the judicial review which is also a form part of the basic Structure.

AMENDMENT AND THE FUNDAMENTAL RIGHTS

Shankari Prasad Case (1951)

Case questioning that whether fundamental rights can be amended under Article 368 & also
questioned the constitutional validity of First amendment act that curtailed the right to
property.

WWW.IASBABA.COM 76
YWM5YTIyN2MzNDVk

IASBABA’S ILP 2022 – POLITY MODULE 1

SC said that Parliament can amend the Fundamental rights under article 368. Also, laws under
Article 13 are ordinary laws & hence can be taken away by Parliament by amendment.

Golak Nath Case (1967)

Case questioned the constitutional validity of putting some state laws under Schedule 9 of
the constitution (by 7th Amendment Act)

Any law put under Schedule 9 is not available for Judicial Review

SC said that its decision in Shankari Prasad Case was wrong & Fundamental rights have an
important position in the constitution & hence can't be amended under article 368.

Constitutional amendment is also a law under Article 13 & hence can't take away the
fundamental rights.

Parliament after this judgment enacted the 24th amendment act, 1971 which amended the
Article 13 & Article 368. The new law stated that Parliament can take away any Fundamental
right by use of Article 368 & such a constitutional amendment act will not be considered as a
"law" under article 13.

Kesavananda Bharati Case (1973)

SC sated that 24th Constitutional amendment act is valid & Parliament can take away
Fundamental rights.

SC at this time came out with doctrine of "basic structure". It states that Parliament cannot
amend the constitution under Article 268 that relates to the change of basic structure of the
constitution i.e. Parliament can't take away those fundamental rights that are a part of basic
structure of the constitution.

Parliament then enacted 42nd Constitutional amendment act, 1976. It states that there is no
limit to the power conferred by Article 368 to the Parliament & any change brought about by
article 368 cannot be questioned in the court of law.

Minerva Mills Case (1980)

The provisions that were laid down by the 42nd Constitutional Amendment act, 1976 were
declared invalid by the Supreme Court in this 1980 Supreme Court case.

Waman Rao Case (1981)

Supreme Court stated that any amendment act can't change the basic structure of the
constitution & these would apply to the amendments after April 24, 1973.

So, as of now, Parliament can do any type of amendment & even amend the Fundamental
rights but which are not changing the "basic structure" of the constitution.

WWW.IASBABA.COM 77
YWM5YTIyN2MzNDVk

IASBABA’S ILP 2022 – POLITY MODULE 1

In I.R.Coelho’s case Supreme Court held that, Art.31-B gives validation based on fictional
immunity. In judging the validity of Constitutional amendment, we have to be guided by the
impact test i.e. Right Test. The basic structure doctrine requires the State to justify the degree
of invasion of Fundamental Rights.

The Parliament is presumed to legislate compatibly with the Fundamental Rights and this is
where judicial review comes in. The greater invasion into essential freedoms, greater is the
need for justification and determination by the Court whether invasion was necessary and if
so to what extent.

The degree of invasion is for the court to decide. Compatibility is one of the species of judicial
review which is premised on compatibility with rights regard as fundamental. The power to
grant immunity, at will, on fictional basis, without full judicial review, will nullify the entire
basic structure doctrine.

Thereby Supreme Court reaffirms the Constitution Supremacy through this basic structure
and now we can say that the “Doctrine of Basic Structure made uncontrolled Constitution
into Controlled one.”

Q. What was held in the Coelho case? In this context, can you say that judicial review is of
key important amongst the basic features of the Constitution? (2016)

Kesavananda Bharati Case

Kesavananda Bharati challenged the Constitution (29th Amendment) Act, 1972, questioning
the Kerala government’s attempts, under two-state land reform acts, to impose restrictions
on the management of its (mutt) property.

He also challenged three Constitutional amendments – the 24th, 25th and 26th amendments
introduced by the Indira Gandhi government.

The principal question that was raised in the case was about the power of Parliament to
amend the Constitution in totality especially with respect to fundamental rights.

The basic question that the Supreme Court had to decide was whether Parliament could alter,
amend, abrogate any part of the Constitution even to the extent of taking away all
fundamental rights?

A 13-judge bench was formed to preside over the case, in which 11 different judgments were
delivered in what is said to be a 7:6 majority.

The Supreme Court then ruled that the ‘basic structure’ of the Constitution was inviolable,
and could not be amended by Parliament. The 'basic structure' doctrine has since been
regarded as a tenet of Indian constitutional law.

WWW.IASBABA.COM 78
YWM5YTIyN2MzNDVk

IASBABA’S ILP 2022 – POLITY MODULE 1

ELEMENTS OF THE BASIC STRUCTURE


The present position is that the Parliament under Article 368 can amend any part of the
Constitution including the Fundamental Rights but without affecting the ‘basic structure’ of
the Constitution.

However, the Supreme Court is yet to define or clarify as to what constitutes the ‘basic
structure’ of the Constitution.

From the various judgments, the following have emerged as ‘basic features’ of the
Constitution or elements / components / ingredients of the ‘basic structure’ of the
constitution:

1. Supremacy of the Constitution


2. Sovereign, democratic and republican nature of the Indian polity
3. Secular character of the Constitution
4. Separation of powers between the legislature, the executive and the judiciary
5. Federal character of the Constitution
6. Unity and integrity of the nation
7. Welfare state (socio-economic justice)
8. Judicial review
9. Freedom and dignity of the individual
10. Parliamentary system
11. Rule of law
12. Harmony and balance between Fundamental Rights and Directive Principles
13. Principle of equality
14. Free and fair elections
15. Independence of Judiciary
16. Limited power of Parliament to amend the Constitution
17. Effective access to justice
18. Principle of reasonableness
19. Powers of the Supreme Court under Articles 32, 136, 141 and 142

Note: No need to remember above 19 points. Just remember the cases and SC rulings.

SIGNIFICANCE OF THE BASIC STRUCTURE


The basic structure limitation comes out of the realization that the only way to safeguard the
Constitution from opportunistic destruction and defilement by temporary majorities in
Parliament is to reject those amendments which go to tarnish its identity.

It arises out of the need to strengthen the Constitution and to prevent its destruction by a
temporary majority in Parliament.

WWW.IASBABA.COM 79
YWM5YTIyN2MzNDVk

IASBABA’S ILP 2022 – POLITY MODULE 1

What is basic structure will depend upon what is vital to Indian democracy and that cannot
be determined except with reference to history, politics, economy and social milieu in which
the Constitution functions.

The Court cannot impose on society anything it considers to be basic. What the judges
consider to be basic structure must meet the requirement of national consciousness about
the basic structure.

Whatever may be the merits or demerits of judicial review, to an extent, the basic structure
limitation upon the constituent power has helped arrest such forces to some extent and to
stabilize the democracy.

CRITIQUES
The doctrine of “basic structure of the Constitution “is very controversial. This doctrine does
not have a textual basis.

We do not find, a provision stipulating that this Constitution has a basic structure and that
this structure is beyond the competence of amending power.

Therefore, the limitation of the amending power through the basic structure of the
Constitution is deprived of positive legal validity.

Moreover, not having its origin in the text of the Constitution, the concept of the “basic
structure of the Constitution” cannot be defined.

What constituted the basic structure of the Constitution? Which principles are or not included
in this concept? An objective and unanimous answer cannot be given to this question.

Constitution may be necessary even to change the original intention of the Constitution
framers, which may not suit a subsequent generation which is to work with the Constitution.
Therefore, to hold that an amendment not falling in the line with the original intention of the
founding fathers is not valid, does not seem to be a sound view.

One of the important critique is that, if the basic structure theory was upheld, “every
amendment made by the Parliament would be subject to judicial approval on the question
whether it damages the core of an essential feature or not… and it is up to the Supreme Court
and High Courts either to validate or invalidate the amendment. It is a step towards the
‘Government of Judges’ as the final say rests with the judges of the Supreme Court not with
the Parliament.

Copyright © by IASbaba
All rights are reserved. No part of this document should be reproduced, stored in a retrieval system
or transmitted in any form or by any means, electronic, mechanical, photocopying, recording or
otherwise, without prior permission of IASbaba.

WWW.IASBABA.COM 80

You might also like